Vision: WWW - Visionias.in

You might also like

Download as pdf or txt
Download as pdf or txt
You are on page 1of 35

VISIONIAS

www.visionias.in
ANSWERS & EXPLANATIONS
ADDITIONAL FULL LENGTH TEST – 3330 (2020)

Q 1.A
• Recent Context: Russia is looking forward to India concluding a new Free Trade Agreement with
the Eurasian Economic Union (EAEU).
• About Eurasian Economic Union: The Eurasian Economic Union is an international organization for
regional economic integration that came into existence on 1st January 2015.
• The EAEU provides for free movement of goods, services, capital, and labor within its borders.
• It pursues, coordinates, & harmonizes the policies in the sectors determined by the Treaty and
international agreements within the Union.
• The member states of the Eurasian Economic Union are:
o The Republic of Armenia,
o The Republic of Belarus,
o The Republic of Kazakhstan,
o The Kyrgyz Republic, and
o The Russian Federation
• Hence option (a) is the correct answer.

Q 2.A
• Surya Sen had participated in the Non-Cooperation Movement and had become a teacher in the national
school in Chittagong. He was imprisoned from 1926 to 1928 for revolutionary activity and afterwards
continued working in the Congress. He was the secretary of the Chittagong District Congress Committee.
He used to say “Humanism is a special virtue of a revolutionary.” He was a lover of poetry and an admirer
of Tagore and Qazi Nazrul Islam.
• Surya Sen decided to organise an armed rebellion along with his associates—Anant Singh, Ganesh
Ghosh and Lokenath Baul—to show that it was possible to challenge the armed might of the mighty
British Empire. Hence, statement 2 is not correct.
• They had planned to occupy two main armouries in Chittagong to seize and supply arms to the
revolutionaries to destroy telephone and telegraph lines and to dislocate the railway link of Chittagong
with the rest of Bengal.
• The raid was conducted in April 1930 and involved 65 activists under the banner of Indian Republican
Army—Chittagong Branch. Hence, statement 1 is correct.
• The raid was quite successful; Sen hoisted the national flag, took salute and proclaimed a provisional
revolutionary government.
• Later, they dispersed into neighbouring villages and raided government targets. Surya Sen was arrested in
February 1933 and hanged in January 1934, but the Chittagong raid fired the imagination of the
revolutionary-minded youth and recruits poured into the revolutionary groups in a steady stream.
Q 3.C
• The Project Monitoring Group (PMG) is an institutional mechanism for the expedited resolution of
issues and regulatory bottlenecks in projects with investments upward Rs. 500 Crores in
India. Hence statement 2 is correct.
• Currently, PMG is situated at Invest India, Department for Promotion of Industry and Internal
Trade (DPIIT), Ministry of Commerce. Hence statement 1 is correct.
• It was set up as a special cell in the Cabinet Secretariat, Government of India in 2013 and was
subsequently brought under the administrative control of the Prime Minister’s Office (PMO) in 2015.
• The relocation was enabled in order to create a one-stop facilitation destination for investors at all stages
of the investment process, including issue resolution. The PMG seeks to enlist the unresolved project
issues in respect of all mid and large-sized Public, Private and ‘Public-Private Partnership’ (PPP) Projects
1 www.visionias.in ©Vision IAS
• DPIIT is mandated as the nodal body for the review of public and private projects facing challenges and
facilitates their resolution through PMG. The issues taken up by PMG are both at Union and State-
Level.
• Related information: Invest India:
o Invest India is the official Investment Promotion and Facilitation Agency of India and acts as the first
point of reference for investors in India.
o It is set up as a non-profit venture under the Department of Industrial Policy and Promotion, Ministry
of Commerce and Industries.

Q 4.B
• Ramanuja, born in Tamil Nadu in the eleventh century, was deeply influenced by the Alvars. Ramanuja
tried to assimilate bhakti to the tradition of the Vedas. Hence, option (b) is not correct.
• He propounded the doctrine of Vishishtadvaita or qualified oneness in that the soul, even when united
with the Supreme God, remained distinct.
• According to him the best means of attaining salvation was through intense devotion to
Vishnu. Vishnu in His grace helps the devotee to attain the bliss of union with Him. He argued that in
order to attain salvation, the grace of God was more important than knowledge about Him.
• Ramanuja emphasized that the path of prapatti or total reliance on, or surrender to God was open to all,
including the Shudras and the Dalits. Thus, Ramanuja tried to build a bridge between the popular
movement based on bhakti, and the upper caste movement based on the Vedas.
• Ramanuja established the Emar Math (Embar Mutt) in Puri and suggested King Choda Ganga Deva to
build a big temple for the deity of Puri-Jagannath, who had been in worship there for several years.
Ramanuja’s influence on the temple is seen in the practice of giving rice as prasada to devotees.

Q 5.A
• Schedules are lists in the Constitution of India that categorize and tabulate bureaucratic activities and
policy of the Government.
• Second Schedule contains the provisions relating to the emoluments, allowances, privileges etc. of the
following:
o The President of India
o The Governors of States
o The Speaker and the Deputy Speaker of the Lok Sabha
o The Chairman and the Deputy Chairman of the Rajya Sabha
o The Speaker and the Deputy Speaker of the Legislative Assembly in the states
o The Chairman and the Deputy Chairman of the Legislative Council in the states
o The Judges of the Supreme Court
o The Judges of the High Courts
o The Comptroller and Auditor General of India. Hence option (a) is the correct answer.
• The provisions in the Second Schedule can be amended by a simple majority of the two Houses of
Parliament outside the scope of Article 368.

Q 6.D
• Recent Context: The 13th Conference of Parties (COP) of the Convention on the conservation of
migratory species of wild animals (CMS), an environmental treaty under the aegis of United Nations
Environment Programme, was hosted by India during 15th to 22nd February, 2020 at Gandhinagar. As a
host, India has been designated the President for the next three years. Ten new species were added
to CMS Appendices at COP13. Seven species were added to Appendix I: the Asian Elephant,
Jaguar, Great Indian Bustard, Bengal Florican, Little Bustard, Antipodean Albatross and the
Oceanic White-tip Shark.
• The Government of India is Signatory to the Convention on Conservation of Migratory wild
Animals (CMS) since 1983. India has signed non-legally binding MOU with CMS on the
conservation and management of Siberian Cranes (1998), Marine Turtles (2007), Dugongs (2008)
and Raptors (2016). like Dugong, under the Conservation and Recovery Action Plan.
• The theme of CMS COP13 in India is, “Migratory species connect the planet and we welcome them
home. “The CMS COP 13 logo is inspired by ‘Kolam’, a traditional artform from southern India. In the
logo of CMS COP-13, Kolam art form is used to depict key migratory species in India like Amur falcon,
humpback whale and marine turtles.
• The mascot for CMS COP13, “Gibi - The Great Indian Bustard” is a critically endangered species which
has been accorded the highest protection status under the Wildlife Protection Act, 1972.
2 www.visionias.in ©Vision IAS
• Related Information on CMS:
o CMS has two Appendices. These appendices list migratory species to which the Convention applies.
▪ Appendix I comprises migratory species that have been assessed as being in danger of extinction
throughout all or a significant portion of their range.
▪ Appendix II covers migratory species that have an unfavourable conservation status and that
require international agreements for their conservation and management, as well as those that
have a conservation status which would significantly benefit from the international cooperation
that could be achieved by an international agreement.
• Hence, option (d) is the correct answer.

Q 7.C
• Recent context: Astronomers of Spain- Institute of Astrophysics of the Canary Islands (IAC) has
discovered an exo-planet WASP-76b where it rains iron. Hence option (c) is the correct answer.
• The conditions of the exo-planet were discovered by Echelle Spectrograph for Rocky Exo-planets and
Stable Spectroscopic Observations (ESPRESSO).
• The high-resolution instrument is installed on the European Southern Observatory's Very Large Telescope
(VLT) in Chile.
• Exoplanet WASP-76b: Exoplanet WASP-76b is located 360 million light-years away, toward
the constellation Pisces. It has days when its temperatures exceed 2,400 degrees Celsius (4,352 degrees
Fahrenheit). The temperature is hot enough to evaporate metals. At nights, the strong winds, cool down
the iron vapor so that it condenses into drops of Iron. The planet always keeps the same face towards its
star as it rotates around it.

Q 8.C
• Pair 1 is correctly matched: The Farakka Barrage is built across the River Ganges in the state of
West Bengal. Farakka Barrage is 2.2 km long and carries a rail-cum-road bridge across the river to
establishes direct road and rail communication links to the North-Eastern States. Canals from the Farakka
Barrage provides a route for the Inland National Waterways 1, from Allahabad to Haldia using the
Ganges, Bhagirathi, and Hooghly river system.
• Pair 2 is not correctly matched: The Prakasam Barrage is located in the Vijayawada in the Krishna
District of Andhra Pradesh. Prakasam Barrage is built across the River Krishna connecting Guntur
and Krishna districts via a 1.2km long river road bridge. Prakasham Barrage also serves the three major
canals run through the entire city and become a tourist attraction of Vijayawada.
• Pair 3 is correctly matched: The Dowleswaram Barrage is built across the River Godavari near
Rajahmundry in the Godavari districts of Andhra Pradesh. Dowleswaram Barrage is the last stretch of
Godavari River before it empties into the Bay of Bengal. The 3.4Km long Barrage is located 8km from
Rajamundry city where the Godavari divided into two streams.
• Other Barrages:
o The Jobra Barrage is built across the River Mahanadi near Cuttack in the Indian state of Orissa.
Jobra Barrage also known as Mahanadi Barrage carries a 2.0 km long road bridge connecting Cuttack
and Jagatpur. The barrage is situated near to the longest river bridge of Orissa and one of the longest
in India, the Mahanadi road bridge.
o The Asan Barrage is located in the famous Doon Valley of Dehradun District near the border of
Uttarakhand and Himachal Pradesh. Asan Barrage built across at the confluence of River Asan and
Eastern Yamuna Canal near Dakpathar.
o The Kota Barrage is built across the River Chambal at Kota City in Rajasthan.
o The Dakpathar Barrage is built across the River Yamuna near to Dakpathar in Uttarakhand state.
o Talwara Barrage is built across the Beas River with big 52 Gates. It is known for world-famous
Pong Dam, Goddess Chintapurni Temple, and Maharana Pratap Sagar lake.

Q 9.C
• The latest generation of Wi-Fi, known as Wi-Fi 6, brings with it some significant performance
improvements that aim to address limitations in older generations. Wi-Fi 6 is also known as "high-
efficiency wireless". Technically, Wi-Fi 6 has a single-user data rate that is 37% faster than 802.11ac, but
what's more significant is that the updated specification offer four times the throughput per user in
crowded environments, as well as better power efficiency which should translate to a boost in device
battery life.
• To achieve those improvements, Wi-Fi 6 implements a variety of changes including several multi-user
technologies which have been borrowed from the cellular industry – namely MU-MIMO and
3 www.visionias.in ©Vision IAS
OFDMA – techniques that greatly improve capacity and performance by enabling more simultaneous
connections and a more thorough use of spectrum. It will ultimately aid in laying a foundation for the
number of nodes expected on upcoming smart infrastructure (e.g. Internet of Things devices).
• Hence both the statements are correct.

Q 10.D
• Unemployment data in India is primarily collected by agencies such as the National Statistical Office
(NSO) of the Ministry of Statistics and Programme Implementation (MOSPI). The government has last
year decided to merge the National Sample Survey Office (NSSO) with the Central Statistics Office
(CSO) leading to the formation of an overarching body - National Statistical Office (NSO).
• Employment-Unemployment Survey (EUS): Employment-Unemployment Survey by erstwhile NSSO
is a comprehensive household survey providing labour force statistics in India. It was first conducted in
1955 and since 1972-73, is conducted every five years. It is the prime source of statistical indicators and
estimates of various parameters of the labour force and activity participation of the population. The main
problem with this survey is its very low frequency and the time lag between the collection and availability
of results. Now, Labour Bureau, an attached office of the Ministry of Labour & Employment releases the
Employment & Unemployment Survey. The survey has been conducted in all the States/UTs by covering
all the districts.
• Periodic Labour Force Survey (PLFS): Considering the importance of the availability of labour force
data at more frequent time intervals, National Statistical Office (NSO) launched Periodic Labour Force
Survey (PLFS) on April 2017. The objective of PLFS is primarily two-fold, one to estimate the key
employment and unemployment indicators (viz. Worker Population Ratio, Labour Force Participation
Rate, Unemployment Rate) in the short time interval of three months for the urban areas only in the
Current Weekly Status (CWS) and to estimate employment and unemployment indicators in both usual
status and CWS in both rural and urban areas annually.
• Employment Exchange: Employment Exchange refers to an office of the central or state government,
which collects and furnishes information on the prospective employers, available vacancies and job
seekers, thereby facilitating job seekers to find a suitable job and for the industry to find the suitable
manpower. Directorate General of Employment and Training (DGET), Ministry of Labour, operates
a National Employment Service (NES) which is entrusted with the matters of Employment
Exchanges. As part of their administrative functioning, Employment Exchanges collect data on various
aspects of employment and unemployment. Data on job seekers registered with the Employment
Exchanges have been analyzed to evaluate the incidence of unemployment in the country.
• Population Census: The census provides information on size, distribution and socio-economic,
demographic and other characteristics of the country's population. Data on main, marginal and non-
workers is collected in Census. According to Census 2011 numbers of the 116 million Indians who were
either seeking or available for work, 32 million were illiterate and 84 million literate. Among literates,
unemployment rates were higher among the better qualified. The ‘unemployed’ included those who were
not currently working but were seeking or available for work, as well as those in marginal employment —
4 www.visionias.in ©Vision IAS
meaning that they worked for fewer than six months in the year preceding the Census — who were
seeking or available for work. Overall, India’s unemployment rate grew from 6.8 p.c. in 2001 to 9.6 p.c. in
2011, based on official Census data. The main problem with this survey is its very low frequency since
census data comes every 10 years.
• Hence, option (d) is the correct answer.

Q 11.D
• Recent Context: In order to make regulatory regime for Nidhi Companies more effective and also to
accomplish the objectives of transparency & investor friendliness in corporate environment of the country,
the Central Government has recently amended the provisions related to NIDHI under the Companies Act
and the Rules (effective from 15.08.2019). The amended provisions of the Companies Act (Section 406)
and Nidhi rules (as amended w.e.f. 15.08.2019) require that the Nidhi companies have to apply to the
Central government for updation of their status/ declaration as Nidhi Company in Form NDH-4.
• Nidhi companies: Under Nidhi Rules, 2014, Nidhi is a company which has been incorporated as a
Nidhi with the object of cultivating the habit of thrift and saving amongst its members, receiving
deposits from, and lending to, its members only, for their mutual benefit. Hence statement 1 is
correct.
• Nidhi Company is a class of Non-Banking Financial Company (NBFC) and Reserve Bank of India (RBI)
has powers to issue directives for them related to their deposit acceptance activities.
• Their core business is borrowing and lending money between their members.
• They are regulated by Ministry of Corporate Affairs. Hence statement 2 is correct.
• They have to comply with two set of norms, one of Public limited company as per Companies Act, 2013
and another is for Nidhi rules, 2014.
• They are registered under the Companies Act, 2013. Hence statement 3 is correct.

Q 12.B
• Article 21 declares that no person shall be deprived of his life or personal liberty except according to
procedure established by law. This right is available to both citizens and non-citizens. Hence statement
1 is not correct.
• In the Gopalan case (1950), the Supreme Court held that the protection under Article 21 is available only
against arbitrary executive action and not from arbitrary legislative action. But, in Menaka case (1978),
the Supreme Court overruled its judgement in the Gopalan case and ruled that the right to life and
personal liberty of a person can be deprived by a law provided the procedure prescribed by that
law is reasonable, fair and just.
• The court held that the ‘right to life’ as embodied in Article 21 is not merely confined to animal existence
or survival but it includes within its ambit the right to live with human dignity and all those aspects of life
which go to make a man’s life meaningful, complete and worth living.
• Under the Constitution, the freedom of movement has two dimensions, viz, internal (right to move inside
the country) and external (right to move out of the country and right to come back to the country).
Article 19 protects only the first dimension. The second dimension is dealt by Article 21 (right to life
and personal liberty). Hence statement 2 is correct.
• Article 359 authorises the president to suspend the right to move any court for the enforcement of
Fundamental Rights during a National Emergency. The 44th Amendment Act of 1978 restricted the
scope of Article 359. Under the Act, during the National Emergency, the President cannot suspend the
right to move the Court for the enforcement of fundamental rights guaranteed by Articles 20 to 21. So
the right to life remains enforceable even during emergency. Hence statement 3 is correct.

Q 13.B
• Kuno-Palpur is situated in the Sheopur district of Madhya Pradesh. This protected region has a core area
of 345 sq km and an outer buffer area of 890 sq km. Together, they constitute the wildlife division,
created in the year 2002 to provide an alternative home to the endangered Asiatic Lion. The undulating
terrain of the Vindhya Range in which the reserve is situated is actually a river valley, created by the
River Kuno, which divides the sanctuary from north to south.
o Three sites, namely Darrah Wildlife Sanctuary, Jawahar Sagar Sanctuary, and Sitamata Wildlife
Sanctuary in Rajasthan and Kuno-Palpur Sanctuary in Madhya Pradesh were identified for lion
introduction. Out of these four, Kuno-Palpur Sanctuary was found to be the most suitable site on
account of its area, shape and vegetation by the Wildlife Institute of India.
o The sanctuary is made up of thick deciduous forest, interspersed with meadows, and
comprises species of trees such as kardhai, gurjan, khair and kahua. Herbivores such as nilgai,
5 www.visionias.in ©Vision IAS
sambar, spotted deer, blackbuck and chinkara, which move in herds, are easily spotted here; while the
carnivores, including panthers, jackals, foxes, hyenas, and bears, can be a little more elusive. Hence
option (b) is the correct answer.
• Bori Wildlife Sanctuary, established in 1977 is located on the foothills of the Satpura range in Madhya
Pradesh. Along with Satpura National Park and Pachmarhi Sanctuary, makes up the Pachmarhi Biosphere
Reserve.
• Nauradehi Wildlife Sanctuary established in the year 1975, it spreads across a total expanse of 1197
square kilometres in the heart of Madhya Pradesh and is shared by four districts, namely Sagar, Damoh,
Narsinghpur and Raisen. The sanctuary’s flora and fauna are sustained by the two rivers basins, Narmada
and Ganga. The sanctuary is thriving in wildlife and is home to exotic canine species including the Indian
Wolf, Striped Hyena and Bengal Fox.
• National Chambal Wildlife Sanctuary, located in Vindhayan ranges is famous for the rare Gangetic
dolphin. The sanctuary was founded in 1979 and is part of a large area co-administered by Rajasthan,
Madhya Pradesh and Uttar Pradesh. Apart from the Gangetic dolphin, the other inhabitants of the
sanctuary include Magar (crocodile) and gharial (alligator), chinkara, sambar, nilgai, wolf and wild boar.
It is the last bastion for some of the country’s most endangered wildlife like the Gharial, Muggar, Turtles,
Otter and the fresh water Dolphin, the Chambal region also boasts of a wide variety of aquatic and
terrestrial bird. Chambal was chosen as one of the main areas for reintroduction the species back into the
wild. The Government of India launched a Crocodile Breeding and Management Project in partnership
with the UNDP/FAO, in 1975. The presence of Gharial prompted the government to establish a protected
area to secure the future of the species. Thus the National Chambal Sanctuary (NCS) was established
covering nearly 610km across the three states.

Q 14.B
• Trade and commerce expanded in India during the Mughal rule in the seventeenth century. The Indian
trading classes were large in numbers, spread out all over thecountry, well organized and highly
professional. Some specialized in long-distance, inter-regional trade, and some in local, retail trade. The
former were called seths, bohra or modi, while the latter was called beoparis or banik.
• The banjaras used to move over long distances, sometimes with thousands of oxen carrying food grains,
pulses, ghee, salt, etc. The more expensive goods, such as textiles, silks, etc., were laden on camels and
mules, or in carts. But it was cheaper to move bulk goods through the rivers on boats. Boat traffic on
waterways and coastal trade along the seashore was more highly developed than now. The trade-in
foodstuffs and a wide range of textile products were the most important components of inter-regional
trade. Hence, statement 2 is correct.
• Merchants in India were not without influence in political quarters where their own interests were
concerned. Thus, each community of merchants had its leader who could intercede with the local officials
on their behalf. We do have instances of strikes (hartal) by merchants in Ahmedabad and elsewhere to
stress their points of view. We have also noted the involvement of members of the Mughal royal family,
and prominent nobles, such as Mir Jumla, in trade. Thus, the Mughal ruling class was not unconcerned
with business and protection of the commercial interests of the country and the trading
classes, though it was not as actively involved in pushing its business interests as some European states
such as Britain, France and Holland were.
• The political stability achieved by the Mughal empire helped create vibrant networks of overland trade
from China to the Mediterranean Sea. Voyages of discovery and the opening up of the New World
resulted in a massive expansion of Asia’s (particularly India’s) trade with Europe. This resulted in a
greater geographical diversity of India’s overseas trade as well as an expansion in the commodity
composition of this trade. An expanding trade brought in huge amounts of silver bullion into Asia to
pay for goods procured from India, and a large part of that bullion gravitated towards India. This
was good for India as it did not have natural resources of silver. As a result, the period between the
sixteenth and eighteenth centuries was also marked by remarkable stability in the availability of metal
currency, particularly the silver rupya in India. This facilitated an unprecedented expansion of minting of
coins and the circulation of money in the economy as well as the ability of the Mughal state to extract
taxes and revenue in cash. Hence, statement 1 is not correct.
Q 15.B
• At the end of the 18th century, the East India Company consolidated its position in most parts of India,
and its activities were closely watched and debated in England. A series of reports were submitted to the
British Parliament on the administration and activities of the East India Company, among these one such
report was the Fifth Report (from the Select Committee of the House of Commons on the East India
Company, released on 28th July 1812).
6 www.visionias.in ©Vision IAS
• This report presented the voice of great opposition from other traders in England and demanded
revocation of the Royal Charter that granted a monopoly to the East India Company of having trade
with India and China.
• Hence option (b) is the correct answer.

Q 16.C
• Recent context: Apollo 13 (50th anniversary) was supposed to make a more challenging landing near the
Fra Mauro.
• About Fra Mauro: The Fra Mauro formation (or Fra Mauro Highlands) is a formation on the near
side of Earth's Moon that served as the landing site for the American Apollo 14 mission in 1971.
Hence option (c) is the correct answer.
o It is named after the 80-kilometer-diameter crater Fra Mauro, located within it. The formation, as well
as Fra Mauro crater, take their names from a 15th-century Italian monk and mapmaker of the same
name.
o Apollo 13 was originally scheduled to land in the Fra Mauro highlands, but was unable due to an in-
flight technical failure.
o Fra Mauro is thought to have been formed from ejecta, or debris, from the impact which
formed Mare Imbrium.
o Conducting experiments at the Fra Mauro formation would have provided greater insights about the
Moon and the Earth’s early geological history.
• About Apollo 13 Mission:
o Apollo 13 was the seventh crewed mission in the Apollo space program and the third meant to
land on the Moon.
o The craft was launched from Kennedy Space Center on April 11, 1970, aboard the Saturn V SA-
508 rocket. But the lunar landing was aborted after an oxygen tank in the service module (SM) failed
two days into the mission.
o The crew instead looped around the Moon, and returned safely to Earth on April 17. While Apollo 13
did not land on the lunar surface, it was able to return photographs that it took when it looped around
the Moon.

Q 17.C
• Pulakesin II, the son of Kirtivarman, was one of the greatest rulers of the Chalukya dynasty. He ruled for
almost 34 years. In this long reign, he consolidated his authority in Maharashtra and conquered large
parts of the Deccan. His greatest achievement was his victory in the defensive war against
Harshavardhana.
• In 620 A.D. Harshavardhana invaded the Chalukya kingdom in the Deccan, which was then ruled
by Pulakesin II. But the Chalukyas’ resistance proved tough for Harshavardhana and he was
defeated. Hence both the statements are correct.
• Pulakesin II was defeated and killed by the Pallava king Narasimhavarman in 642 A.D. His son
Vikramaditya, who was also as great a ruler as his father, succeeded him. He renewed the struggle against
his southern enemies. He recovered the former glory of the Chalukyas to a great extent.

Q 18.D
• Recent context: Army gets upgraded 155mm artillery gun at Defexpo.
• About Sharang artillery Gun :
o Sharang is the 130mm artillery gun ‘up-gunned’ to 155mm, 45 calibre up-gunning based on the
Army’s tender. Hence pair 1 is correctly matched.
o The gun’s range has now gone from 27 km to over 36 km with the upgrade.
o It also has more explosive capability and hence and more damage potential.
o In all, OFB will upgrade 300 130mm guns to 155mm and the contract will be completed in four
years.
• Recent context: Indian Army successfully test-fires Spike LR missile (long-range). Hence pair 2 is
correctly matched.
• About Spike:
o Spike anti-tank missiles is a 4th generation missile which can engage any target with precision at
ranges up to 4 km.
o It is developed and designed by Israel’s Rafael Advanced Defense Systems.
o The missile is man-portable and has its own vehicle-launch and helicopter-launched variants.
o It will bolster Indian Army’s fire power capability.
7 www.visionias.in ©Vision IAS
• Recent context: India will buy 24 MH-60 Romeo helicopters from Lockheed Martin for the Indian
Navy for USD 2.6 billion.
• About Mh 60 Romeo:
o These helicopters are considered to be the world's most advanced maritime helicopter. Hence pair
3 is correctly matched.
o The proposed sale will provide India the capability to perform anti-surface and anti-submarine
warfare missions.
o It will also enhance the Navy’s ability to perform secondary missions, including search and rescue,
and communications relay.

Q 19.C
• Recently the Environment Minister virtually launched piloting of the National Transit Pass System. The
National Pass System enhances seamless movement of forest produce. Hence option (c) is the
correct answer.
• It is an online system for issuing transit permits for timber, bamboo and other forest produce. NTPS will
bring ease of business and expedite the issuance of transit permits for timber, bamboo and other minor
forest produce without physically going to forest offices. Through mobile application, e-pass will be
issued for the transit of forest produce. The pilot project will be functional in Madhya Pradesh and
Telangana for now.
• People can apply for Passes from their mobile phones as well as receive e passes in their mobile phones
This will enhance the seamless movement of forest produce.

Q 20.C
• The various committees and commissions which have examined our electoral system, election machinery
as well as election process are
• Indrajit Gupta Committee on State Funding of Elections (1998):
o Only Election Commission recognized political parties should be given the state support in terms of
printing material and facilities; electronic media time; vehicles and fuel etc.
o Complete account of the election expenditure should be filled by the parties to the ECI.
o Political parties should compulsorily submit their annual accounts to the income tax department
• Dinesh Goswami Committee
o Time limit for bye-elections.
o Increase in deposits from independents.
o The committee also called for amendment of the anti-defection law to restrict disqualification.
• Tarkunde Committee (1975):
o The election commission should be a three member body.
o The minimum age of voting should be 18 years.
o The TV and radio should be placed under the control of autonomous statutory corporation.
o A voter’s council should be formed in as many constituencies as possible which can help in free and
fair election.
• Also various reports of Law Commission, National Commission to Review the Working of the
Constitution proposed several electoral reforms.
• Both Abhijit Sen Committee and Narasimhan Committee are related to reforms in financial sector.
• Hence option (c) is the correct answer.

Q 21.A
• Tamil has a literary tradition that goes back to the period of the Sangams (c. 300 BC – c. 300 CE). The
most well-known surviving works from this period are the Tolkappiyam (a book of grammar attributed to
Tolkappiyar), Ettu Togai (Eight Anthologies), Pattu Pattu (Ten Idylls), epics like Silappadikaram and
Manimekalai (authored by Ilango Adigal and Sattanar respectively), Thiru Kural (a Tamil treatise par
excellence by Thiruvalluvar) and Perungadai (another work of epic proportion by Kongu Velir).
• This period was succeeded by the age of devotional poetry of both the Shaiva and the Vaishnava
variety in Tamil literature. These works are referred to as the Thevaram and the Nalayira Divya
Prebandham respectively. Hence option (a) is the correct answer.

Q 22.C
• Statement 1 is correct: Sweat comes out of three types of glands distributed across all over our
skin, secreting water and substances that help keep our body at the optimum temperature of 37
degrees C (or 98.4 degrees F). It is 99% water containing sodium, potassium, calcium, magnesium and
8 www.visionias.in ©Vision IAS
chloride ions, ammonium ions, urea, lactic acid, glucose and other minor components. Our brain has
temperature-sensitive nerve cells (neurons) which control the sweat glands in releasing the fluid
depending on the temperature and physical and metabolic activity of the body. Sweat is thus our body’s
thermo-regulator.
• Statement 2 is correct: An analysis of the sweat fluid in a patient and how it compares with that of a
‘normal’ individual will thus be of diagnostic value (just as much as other body fluids do). For
example, in the illness called cystic fibrosis, the ratio of the sodium to chloride ions in the sweat is
different from that of a normal individual. Likewise, the amount of glucose in the sweat of a diabetic is
higher than normal. But the problem is the amount of sweat available from the skin. Recently, e-skin
patches have been developed for real-time measurements of some chosen component in the sweat
• Statement 3 is correct: In humans, sweat is hypoosmotic relative to plasma (i.e. less concentrated).
Sweat is found at moderately acidic to neutral pH levels, typically between 4.5 and 7.0.

Q 23.B
• Recent context: Prime Ministers of India and Nepal have jointly inaugurated the Motihari-
Amlekhganj petroleum product pipeline, the first of its kind in South Asia.
• About the Pipeline:
o The 69 km-long pipeline from Motihari in Bihar to Amlekhganj in Nepal is the first-ever cross-
border petroleum product pipeline in the South Asia region. Hence option (b) is the correct
answer.
• Significance of the pipeline: As of now, tankers carry petroleum products from India to Nepal as part of
an arrangement which is in place since 1973. Every year, the pipeline will carry two million metric tonne
of clean petroleum products at an affordable price to Nepal. It will help to enhance the energy security of
the region and substantially cut down on transit costs.

Q 24.A
• Statements 1 and 2 are correct: Ammonium nitrate is a chemical compound with the chemical
formula NH4NO3. It is a white crystalline solid consisting of ions of ammonium and nitrate. It is
highly soluble in water and hygroscopic as a solid, although it does not form hydrates. It is
predominantly used in agriculture as a high-nitrogen fertilizer.
• Its other major use is as a component of explosive mixtures used in mining, quarrying, and civil
construction. Mining companies led by Coal India Ltd are the biggest consumers of ammonium nitrate,
the chemical compound in India. Ammonium nitrate is imported in bulk and in large quantities through
Central government-run Visakhapatnam Port Trust by some 15 explosive manufacturers located in the
eastern states including the mineral belts of Orissa and Bihar. The chemical is used to manufacture
explosives that are in turn sold to miners who use it for blasting during mining operations.
• Statement 3 is not correct: Miners source explosives from manufacturers holding permits from
Petroleum and Explosives Safety Organisation (PESO), to import the chemical. PESO is a regulatory
agency attached to the Department for Promotion of Industry and Internal Trade under the Ministry of
Commerce and Industries, to administer Explosives Act 1884, Petroleum Act 1934, Inflammable
Substances Act 1952, Ammonium Nitrate Rules 2012 and Environment Protection Act 1986, among
others, for controlling import, export, transport, storage and usage of explosive
materials, flammable materials, pressure vessels, cryogenic vessels, design and installation of all
necessary and relevant infrastructure. The Central government has tightened the import, storage, transport
and manufacture of explosives using the chemical by notifying the Ammonium Nitrate Rules, 2012.

Q 25.B
• Snow leopard (Panthera uncial) is large cat native to mountain ranges of Central and South Asia-
including the Himalayas, and Russia’s remote Altai mountains. The snow leopard inhabits the higher
Himalayan and trans-Himalayan landscape in the five states of Jammu and Kashmir, Himachal Pradesh,
Uttarakhand, Sikkim, and Arunachal Pradesh. This area contributes to about 5% of the global snow
leopard range.
• Snow leopards are categorized as ‘Vulnerable’ by IUCN and in the Schedule I of the Indian
Wildlife (Protection) Act 1972. They are listed in Appendix I of the Convention on International Trade
in Endangered Species (CITES) and the Convention on Migratory Species (CMS). The IUCN in 2017 has
down-listed the snow leopard on the Red List of Threatened Species from ‘Endangered’ to
‘Vulnerable’. Hence, statement 1 is not correct.
• India’s first Snow Leopard Conservation Centre will be opened in Uttarkashi forest division in
Uttarakhand. The conservation centre will be built by the Uttarakhand forest department along with
9 www.visionias.in ©Vision IAS
United Nations Development Programme (UNDP) as part of its six-year-long project, SECURE
Himalayas. The project aims at securing livelihoods, conservation, sustainable use and restoration of high
range Himalayan ecosystems. Hence, statement 2 is correct.

Q 26.B
• Superconductivity refers to complete disappearance of electrical resistance in various solids when they
are cooled below a characteristic temperature. This temperature, called the transition temperature,
varies for different materials but generally is below 20 K (−253 °C). Hence statement 1 is not correct.
• The superconducting material shows some extraordinary properties which make them very important for
modern technology.
o In Superconducting state, the superconducting material shows the zero electric resistance (infinite
conductivity).
o A Superconductor, when it is cooled below the critical temperature Tc), expel the magnetic field
and doesn’t allow the magnetic field to penetrate inside it. This phenomenon in superconductors is
called Meissner effect.
• Superconductors are used in the following applications:
o Maglev (magnetic levitation) trains: These work because a superconductor repels a magnetic field
so a magnet will float above a superconductor – this virtually eliminates the friction between the train
and the track. Hence statement 2 is correct.
o Superconductors are used in Large hadron collider or particle accelerator. to make extremely powerful
electromagnets to accelerate charged particles very fast (to near the speed of light).
o SQUIDs (Superconducting QUantum Interference Devices) are used to detect even the weakest
magnetic field. They are used in mine detection equipment to help in the removal of land mines.

Q 27.A
• Time Capsule is a container of any size or shape, which accommodates documents, photos and
artefacts typical of the current era and is buried underground, for future generations to
unearth. The time capsule requires special engineering so that the contents don’t decay, even if pulled out
after a century. Material such as aluminium and stainless steel are used for the encasing, and documents
are often reproduced on acid-free paper. Hence, option (a) is the correct answer.
• While the term “time capsule” was coined in the 20th century, among the earliest examples of one dates
back to 1777, found by historians inside the statue of Jesus Christ in a church in Spain during restoration
work in December 2017. Are there any time capsules in India?
• There have been a number of prominent examples. One time capsule, outside the Red Fort and placed
underground in 1972 by Prime Minister Indira Gandhi, was dug out by the subsequent government. Other
time capsules are at a school in Mumbai, IIT-Kanpur, Lovely Professional University in Jalandhar, and
Mahatma Mandir in Gandhinagar.
Q 28.C
• Gandhi travelled incessantly during 1929 preparing people for direct political action - telling the youth to
prepare for the fiery ordeal, helping to organise constructive work in villages and redressing specific
grievances.
• The Working Committee of the Congress in 1929 appointed the following persons with power to co-
opt, to constitute the Foreign Cloth Boycott Committee:
o Mahatma Gandhi (Chairman),
o Motilal Nehru
o Madan Mohan Malaviya
o Moulana Abul Kalam Azad
o M.A. Ansari
o Jawaharlal Nehru
• The Committee was required to formulate a plan to bring about boycott of foreign cloth by advocating and
stimulating production and adoption of handspun and handwoven Khaddar. Gandhi initiated the
campaign in March 1929 in Calcutta and was arrested. This was followed by bonfires of foreign cloth
all over the country

Q 29.D
• Telecom Disputes Settlement and Appellate Tribunal (TDSAT) was set up under the provision of
Telecom Regulatory Authority of India Act (TRAI) to adjudicate disputes and dispose of appeals with a
view to protect the interests of service providers and consumers of the telecom sector and to promote and
ensure orderly growth of the telecom sector. Hence, statement 1 is correct.
10 www.visionias.in ©Vision IAS
• Composition and Qualification of Members: The Tribunal consists of a Chairperson and two Members
appointed by the Central Government. The Chairperson should be or should have been a Judge of the
Supreme Court or the Chief Justice of a High Court. A Member should have held the post of Secretary
to the Government of India or any equivalent post in the Central Government or the State Government for
a period of not less than two years or a person who is well versed in the field of technology,
telecommunication, industry, commerce or administration. Hence, statement 2 is not correct.
• Powers and Jurisdiction:
o The Tribunal exercises jurisdiction over Telecom, Broadcasting, IT and Airport tariff matters under
the TRAI Act, 1997 (as amended), the Information Technology Act, 2008 and the Airport Economic
Regulatory Authority of India Act, 2008.
o The Tribunal exercises original as well as appellate jurisdiction in regard to Telecom, Broadcasting,
and Airport tariff matters. In regard toCyber matters the Tribunal exercises only the appellate
jurisdiction. Hence, statement 3 is correct.
• In respect of Telecom, Broadcasting, and Airport tariff matters, the Tribunal's orders can be appealed to
the Supreme Court but only on substantial questions of law. However, no appeal lies against an
interlocutory order or against any decision or order made by the Tribunal with the consent of the parties.
In regard to Cyber matters, the Tribunal’s order can be appealed before the High Court.

Q 30.D
• Koli is performed in Maharashtra. Since the dance is performed by fishermen, it involves elements of
fishing and sea. Both men and women are involved in this interesting dance form which is a treat to
watch. Hence pair 1 is correctly matched.
• Dumhal, the age-old dance, is still kept alive by the Rauf tribe of Jammu & Kashmir and is performed
by men who wear long and colorful robes, accompanied by tall conical caps. The performers place a
banner into the ground at a fixed location, and the dance is performed around this banner. Hence pair 2 is
not correctly matched.
• Wangala or Wangla is the most popular festival among the Garos tribe residing mainly in (Meghalaya as
well as in Nagaland and Assam). This is celebrated after harvest to thank Misi Saljong, the ‘Sun God’ and
Mini-ma Kiri Rokime, the ‘Goddess of Grains’. This festival is usually celebrated in the month of October
or November accompanied by customary rituals, merry making, and dancing for seven consecutive days
and nights or till food and drinks last. Dressed in the best of their traditional attires and ornaments, both
the young and old males and females in the Garo villages heartily take part in this festival. Hence pair 3
is correctly matched.
Q 31.C
• The Wallace Line or Wallace's Line is a faunal boundary line drawn in 1859 by the British naturalist
Alfred Russel Wallace and named by English biologist Thomas Henry Huxley that separates the
Biogeographical realms of Asia and Wallacea, a transitional zone between Asia and Australia.
• West of the line are found organisms related to Asiatic species; to the east, a mixture of species of Asian
and Australian origin is present.
• The northern part of the line was altered by T.H. Huxley to fall to the west of the Philippines (excluding
Palawan). Huxley’s line is considered a more appropriate delineation by some zoogeographers because
the Philippines has a highly idiosyncratic fauna.
• Lydekker's Line is a biogeographical boundary through Indonesia that separates Wallacea on the west
from Australia-New Guinea on the east. It follows the edge of the Sahul Shelf, an area from New Guinea
to Australia of shallow water with the Aru Islands on its edge.
• Along with Wallace's Line and Huxley's Line, it indicates the definite effect of geology on the
biogeography of the region, something not seen so clearly in other parts of the world.
• Hence option (c) is the correct answer.

Q 32.C
• The British in India, included the definitions of everyday Indian administrative terms in every rule and
law, for the understanding of the British Officials and governance. Some of these terms were -
• Jotedar - They were rich ryots. Ryots in Bengal did not cultivate the land directly but leased it out to the
under-ryots. At the end of the 18th century when most of the zamindars were facing crisis
the jotedars consolidated their position in villages and became identical with the mandals. They
controlled local trade as well as money lending.
• Mandals - They were village headmen. They had effectual influence and authority over the officers in
their village.

11 www.visionias.in ©Vision IAS


• Amlah - An officer of the zamindar, appointed by the court, who visited villages at the time of rent
collection to resolve and settle any dispute in the assessment and collection of rent.
• Adhiyars - A large part of the land of jotedars was cultivated through sharecroppers (adhiyars or
bargadars) who brought their own ploughs, laboured in the field, and handed over half the produce to the
jotedars after the harvest.
• Hence only options 3 and 4 are correctly matched.

Q 33.A
• The 101st Constitutional Amendment Act, 2017 to the Constitution introduced GST in India. It
amended the Constitution to give concurrent powers to Parliament and state legislatures to levy a Goods
and Services Tax (GST). Hence pair 2 is not correctly matched.
• 102nd Constitutional Amendment Act, 2018 provides constitutional status to the National
Commission for Backward Classes (NCBC). It has the authority to examine complaints and welfare
measures regarding socially and educationally backward classes. Previously NCBC was a statutory body
under the Ministry of Social Justice and Empowerment.
• The 103rd Constitutional Amendment Act, 2019 provides reservation for Economically Weaker
Sections (EWS) amends Articles 15 and 16 of the Constitution to provide for reservation benefit in
economically weaker sections. Part III of the Indian Constitution deals with fundamental rights. While the
103rd Constitutional amendment act does include the word ‘reservation’ in Part III of the Indian
Constitution, it is not the first one to do so. The word ‘reservation’ under Part III of the Indian
Constitution has already been included under Article 16 (Article 16(4), Article 16(4A) and Article
16(4B)).
• The 103rd Constitutional amendment sets an upper limit of 10 percent for reservation to
economically weaker sections in the Constitution itself. This amendment provides reservation over
and above 50%. Hence pair 3 is correctly matched.
• 104th Constitutional Amendment Act, 2020 extended the reservation of seats for SCs and STs in the
Lok Sabha and states assemblies from seventy years to eighty. It removed the reserved seats for the
Anglo-Indian community in the Lok Sabha and state assemblies. Hence pair 1 is correctly matched.

Q 34.B
• NITI Aayog in collaboration with International Transport Forum (ITF) has launched the
“Decarbonising Transport in India” project on 24 June, with the intention to develop a pathway towards a
low-carbon transport system for India. Hence, statement 1 is not correct.
• The ITF is an intergovernmental organisation for transport policy of which India has been a member since
2008
• The “Decarbonising Transport in India” project will design a tailor-made transport emissions assessment
framework for India. It will provide the government with a detailed understanding of current and future
transport activity and the related CO2 emissions as a basis for their decision-making. Hence, statement 2
is correct.
• The India project is a part of the ITF’s wider initiative of decarbonising transport and is a part of DTEE
(Decarbonising Transport in Emerging Economies) series of projects that are aimed at lowering carbon
emissions across different regions in the world.

Q 35.C
• The Galwan River flows from the Chinese occupied Aksai Chin region administered by China to
the Ladakh union territory of India. It originates to the east of the area of Samzungling on the
eastern side of the Karakoram range and flows west to join the Shyok River. Hence the correct
option is (c).
• The recent clashes between the Indian army and the Chinese PLA occurred in this Galwan Valley due to
intrusions by the Chinese soldiers. And Chinese opposition to Darbuk–Shyok-Daulat Beg Oldi
Road that runs along the Shyok River and almost parallel to Line of Actual Control but well within the
Indian territory.
• The river is named after Ghulam Rasool Galwan, a Ladakhi explorer of Kashmiri descent, who first
explored the course of the river. In 1899, he was part of a British expedition team that was exploring the
areas to the north of the Chang Chenmo valley, when he ran into this previously unknown river valley.
• Nubra River is another tributary of Shyok River and the Nubra Valley is also of strategic importance
as it leads to Siachen Glacier.
• River Shyok and Sutlej are tributaries of River Indus.

12 www.visionias.in ©Vision IAS


Q 36.D
• Article 32 provides the right to Constitutional remedies which means that a person has right to move
to Supreme Court (and high courts also) for getting his fundamental rights protected. While
Supreme Court has power to issue writs under article 32, High Courts have been given same powers under
article 226.
• It contains the following four provisions:
o The right to move the Supreme Court for the enforcement of the Fundamental Rights.
o The Supreme Court shall have power to issue directions or orders or writs for the enforcement of
any of the fundamental rights.
o Parliament can empower any other court to issue directions, orders and writs of all kinds.
o The right to move the Supreme Court shall not be suspended except as otherwise provided for by the
Constitution. Thus the Constitution provides that the President can suspend the right to move any
court for the enforcement of the fundamental rights during a national emergency (Article 359).
• The right to get the Fundamental Rights protected is in itself a fundamental right under Article 32. This
makes the fundamental rights real. That is why Dr. Ambedkar called Article 32 as the most important
article of the Constitution–‘an Article without which this constitution would be a nullity. It is the very
soul of the Constitution and the very heart of it'.
• Hence option (d) is the correct answer.

Q 37.A
Passing of the Appropriation and Finance Bill:
• The constitution (Article 114) states that no money shall be withdrawn from the Consolidated Fund
of India except under appropriation made by law.
• Accordingly an appropriation bill is introduced to provide for the appropriation , out of Consolidated Fund
of India;
o the grants voted by the Lok Sabha
o The expenditure charged on the Consolidated Fund of India.
• The Finance Bill is introduced to give effect to financial proposals of the Government of India for the
following year . It is subjected to all the conditions applicable to the money bill.
• No amendments can be proposed to the appropriation bill in either house of the parliament that will
have the effect of varying the amount or altering the destination of any grant voted, or of varying
the amount of any expenditure charged on the Consolidated Fund of India. Unlike the
appropriation bill the amendments ( seeking to reject or reduce a tax) can be moved in the case of
finance bill. Hence statement 2 is not correct.
• The appropriation bill becomes appropriate Act after it is assented to by the president . This act
authorise( or legalise ) the payments from the the Consolidated fund of India. The Finance Act
legalise the income side of the budget and complete the process of the enactment of the budget.
Hence statement 1 is correct.
• According to the provisional Collection of Taxes Act of 1931, the Finance Bill must be enacted ( i.e.
passed by the Parliament and assented to by the President) within 75 days.

Q 38.D
• Sher Shah Suri (1540-1545) was an Afghan leader who took over the Mughal Empire after defeating
Humayun in 1540.
• Sher Shah occupied the throne of Delhi for not more than five years, but his reign proved to be a landmark
in the Sub-continent. As a king, he has several achievements in his credit.
o He established an efficient public administration.
o He set up a revenue collection system based on the measurement of land.
o Justice was provided to the common man.
o Numerous civil works were carried out during his short reign; planting of trees, wells and building of
Sarai (inns) for travellers was done.
o Roads were laid; it was under his rule that the Grand Trunk road was built.
• However, Sher Shah did not survive long after his accession on the throne and died in 1545 after a
short reign of five years. After his death, Humayun was able to defeat his successor, Sikandar Suri
and regain the crown of the Hindustan.

13 www.visionias.in ©Vision IAS


Q 39.A
• Nitrogen is the most abundant gas in the atmosphere. It plays an important role in slowing the rate of
burning and controlling the combustion, in respiration and other biological and physical
processes. Hence statement 1 is correct.
• Nitrogen is largely used as nitrogen fertilisers in agriculture. Nitrogen fertilizers are manufactured on
a large scale from the nitrogen contained in the atmosphere.
• Atmospheric nitrogen cannot be used directly by plants without undergoing a transformation. To
be used by plants, the Nitrogen must be transformed through a process called nitrogen fixation.
Hence statement 2 is not correct.
• Nitrogen is so vital because it is a major component of chlorophyll, the compound by which plants use
sunlight energy to produce sugars from water and carbon dioxide (i.e., photosynthesis). Hence statement
3 is correct.
Q 40.D
• Ocean deposits usually consist of unconsolidated sediments, which can come from various sources, and
are deposited at the ocean floor. Thickness of these deposits vary greatly from one ocean to another. They
can be differentiated into following categories.
• The Muds: These are terrigenous deposits because they are derived from land and mainly deposited on
the continental shelves. The muds are referred to as blue, red and green muds; their colouring depends on
their chemical content. Hence pair 2 is correctly matched.
• The Oozes: These are pelagic deposits because they are derived from the ocean. They are made of shelly
and skeletal remains of marine microorganisms with calcareous and siliceous parts. Oozes have a very
fine and flour like texture and either occur as accumulated deposits or float about in suspension. Hence
pair 1 is correctly matched.
• The Clays: These occur mainly as red clays in deeper parts of the ocean. Red clay is an accumulation of
volcanic dust blown out from volcanoes during a volcanic eruption. Hence pair 3 is correctly matched.

Q 41.A
• Article 244 in Part X of the Constitution envisages a special system of administration for certain areas
designated as ‘scheduled areas’ and ‘tribal areas’. The Sixth Schedule of the Constitution deals with the
administration of the tribal areas in the four northeastern states of Assam, Meghalaya, Tripura and
Mizoram.
• The features of administration contained in the Sixth Schedule are as follows:
o The Governor is empowered to organise and re-organise the autonomous districts. Thus, he can
increase or decrease their areas or change their names or define their boundaries. Hence statement 1
is not correct.
o Each autonomous district has a district council consisting of 30 members, of whom four are
nominated by the governor and the remaining 26 are elected on the basis of adult franchise.
Hence statement 2 is not correct.
o The district and regional councils administer the areas under their jurisdiction.
o The district and regional councils within their territorial jurisdictions can constitute village councils or
courts for trial of suits and cases between the tribes.
o The Council can also make regulations for the control of money lending and trading by nontribals.
But, such regulations require the assent of the governor.
o The district and regional councils are empowered to assess and collect land revenue and to impose
certain specified taxes. Hence statement 3 is correct.
o The acts of Parliament or the state legislature do not apply to autonomous districts and autonomous
regions or apply with specified modifications and exceptions.
Q 42.B
• In September 1946, the Bengal Provincial Kisan Sabha gave a call to demand - Tebhaga - two-thirds’
share - to the bargardars, the share croppers instead of the one-half share. The storm centre of the
movement was north Bengal. The central slogan was “nij khamare dhan tolo”- i.e., sharecroppers taking
the paddy to their own threshing floor and not to the jotedar’s house, as before, so as to enforce
tebhaga. Hence statement 1 is not correct and statement 2 is correct.
• The share croppers were encouraged by the fact that the Bengal Land Revenue Commission, popularly
known as the Floud Commission, had already made this recommendation.
• The movement received a great boost in late January 1947 when the Muslim League Ministry led by
Suhrawardy published the Bengal Bargadars Temporary Regulation Bill. The Muslim League Ministry
failed to pursue the bill in the Assembly and it was only in 1950 that the Congress Ministry passed a
Bargadars Bill which incorporated, in substance, the demands of the movement.
14 www.visionias.in ©Vision IAS
Q 43.C
• The real exchange rate (RER) measures the value of currencies, taking into account changes in the price
level. The real exchange rate shows what you can actually buy.
• While two currencies may have a certain exchange rate on the foreign exchange market, this does not
mean that goods and services purchased with one currency cost the equivalent amounts in another
currency. This is due to different inflation rates with different currencies. Real exchange rates are thus
calculated as a nominal exchange rate adjusted for the different rates of inflation between the two
currencies.
• A high RER implies that foreign goods are relatively cheap and domestic goods are relatively
expensive. Whereas a low RER implies that foreign goods are relatively expensive and domestic goods
are relatively cheap. Hence, statement 1 is correct.
• The RER is just a relative price. This means that the relative demand for domestic and foreign goods is
affected by the relative price of two sets of goods. If initially, RER is low, domestic residents will buy few
imported goods. For exactly the same reason, foreigners will want to buy more domestic. Therefore, net
exports will be high.
• If RER is high, domestic residents (say Indians) will want to buy many imported goods and foreigners
will want to buy a few Indian goods. So the number of net exports demanded will be very much on the
low side and imports rise due to cheaper prices. Hence, statement 2 is correct.

Q 44.C
• In March 1940, Subhas Chandra Bose convened an Anti-Compromise Conference at Ramgarh. It
was a joint effort of the Forward Bloc and the Kisan Sabha.
• It was resolved at the conference that a world-wide struggle should be launched on April 6, the first day of
the National Week, with a call to the people not to help the Imperialist War (2nd World War) with any
resource—men, money or materials.
• He called for resistance to be offered to all forms of exploitation of Indian resources for the imperial
cause. There was enthusiastic participation by the people in the struggle launched on April 6.

Q 45.D
• A Joint Liability Group (JLG) is an informal group comprising of 4-10 individuals coming together
for the purpose of availing bank loan on individual basis or through group mechanism against the
mutual guarantee.
• Generally, the members of a JLG would engage in a similar type of economic activity. In certain groups,
members may prefer to undertake a different type of economic activities as well. The members would
offer a joint undertaking to the bank to enable them to avail loans. JLG members are expected to provide
support to each other in carrying out occupational and social activities.
• They are typically credit groups of small or marginal tenant farmers who do not have a proper title
of their farmland. Through these JLGs, banks plan to extend assistance to the financially excluded
sections of the society, especially landless farmers.
• Objectives of Joint Liability Groups (JLGs):
o To augment the flow of credit to landless farmers cultivating land as tenant farmers, oral lessees or
sharecroppers and small/marginal farmers as well as other poor individuals taking up farm activities,
off-farm activities and non-farm activities.
o To serve as a collateral substitute for loans to be provided to the target group.
o To build mutual trust and confidence between the bank and the target group.
o To minimize the risks in the loan portfolio for the banks through group approach, cluster approach,
peer education and credit discipline.
o To provide food security to vulnerable section through enhanced agriculture production, productivity
and livelihood promotion through joint farming and cluster approach.
• Features of JLG and criteria for membership:
o A JLG is an Informal group in which 4 - 10 individuals come together and form a group
o All the members agree and avail the loan in the group through a formal agreement
o Members should belong to similar socio-economic status, background and environment
carrying out farming and Allied activities and who agree to function as a joint liability
group. This way the groups would be homogeneous and organized by like-minded farmers/
Individuals and develop mutual trust and respect.
o The members should be residing in the same village/ area/ neighbourhood and should know and trust
each other well enough to take up joint liability for group/individual Loans.

15 www.visionias.in ©Vision IAS


o Members who have defaulted to any other formal financial institution, in the past, are debarred from
the Group Membership.
• Loans to individual farmers [including Self Help Groups (SHGs) or Joint Liability Groups (JLGs)],
directly engaged in Agriculture and Allied Activities, viz., dairy, fishery, animal husbandry, poultry, bee-
keeping and sericulture are treated as eligible for classification under priority sector target. Hence, option
(d) is not correct.
Q 46.D
• Mesolithic or Middle Stone Age falls roughly from 10000 B.C. to 6000 B.C. It was the transitional phase
between the Paleolithic Age and Neolithic Age.
• Mesolithic remains in India are found in Langhnaj in Gujarat, Adamgarh in Madhya Pradesh and also in
some places of Rajasthan, Uttar Pradesh and Bihar. The paintings and engravings found at the rock
shelters give an idea about the social life and economic activities of Mesolithic people.
• In the sites of Mesolithic Age, a different type of stone tools is found. These are tiny stone artifacts, often
not more than five centimeters in size, and therefore called microliths.
• People did not use to build permanent settlements, but only short-period seasonal settlements. They fed
themselves on hunting, picking up herbs, and lived in smaller movable groups, wandering after food or
raw material at long distances. Hence, statements 1 is not correct.
• However, there seems to have been a shift from big animal hunting to small animal hunting and fishing.
The use of bow and arrow also began during this period.
• Mesolithic people used to wear clothes made up of bark and leaves in the summers and in the winters they
used to wear animal skins. Hence, statement 2 is not correct.
• Also, there began a tendency to settle for longer periods in an area. Therefore, domestication of animals,
horticulture and primitive cultivation started. Animal bones are found in these sites and these include dog,
deer, boar and ostrich. Occasionally, burials of the dead along with some microliths and shells seem to
have been practiced.
• The chief characteristic features of the Neolithic culture are the practice of agriculture, domestication of
animals, polishing of stone tools and the manufacture of pottery. In fact, the cultivation of plants and
domestication of animals led to the emergence of village communities based on sedentary life. The
people of Neolithic Age used clothes made of cotton and wool.

Q 47.C
• Since the mid-20th Century, average global temperatures have warmed about 0.6°C (1.1°F), but the
warming has not occurred equally everywhere.
• Temperatures have increased about twice as fast in the Arctic as in the mid-latitudes, a
phenomenon is known as “Arctic amplification. Hence option (c) is the correct answer.
• Why are temperatures warming faster in the Arctic than the rest of the world?
o The loss of sea ice is one of the most cited reasons.
o When bright and reflective ice melts, it gives way to a darker ocean; this amplifies the warming trend
because the ocean surface absorbs more heat from the Sun than the surface of snow and ice. In more
technical terms, losing sea ice reduces Earth’s albedo: the lower the albedo, the more a surface
absorbs heat from sunlight rather than reflecting it back to space.
• However, other factors contribute as well. Thunderstorms, for instance, are much more likely to occur in
the tropics than the higher latitudes. The storms transport heat from the surface to higher levels of the
atmosphere, where global wind patterns sweep it toward higher latitudes. The abundance of thunderstorms
creates a near-constant flow of heat away from the tropics, a process that dampens warming near the
equator and contributes to Arctic amplification.
Q 48.A
• The greatest ruler of the Pratihara dynasty was Mihir Bhoja. He recovered Kannauj (Kanyakubja) by
836, and it remained the capital of the Pratiharas for almost a century. He built the city Bhojpal
(Bhopal). Raja Bhoja and other valiant Gujara kings faced and defeated many attacks of the Arabs from
west. Hence statement 1 is correct.
• Between 915-918 A.D, Kannauj was attacked by a Rashtrakuta king, who devastated the city leading to
the weakening of the Pratihara Empire. In 1018, Kannauj then ruled by Rajyapala Pratihara was
sacked by Mahmud of Ghazni. The empire broke into independent Rajput states. Hence statement 2 is
not correct.
• Muhammad Ghori, the Sultan of the Ghurid Empire, between the duration of 1173-1206AD, reigned over
the region of modern-day Afghanistan, Bangladesh, Tajikistan, Pakistan, Iran, and Northern India. In
1191, passing through the Kyber Pass, Ghori set out to invade Indian subcontinent by first capturing a fort
in Bhatinda that belonged to Prithvi Raj Chauhan.
16 www.visionias.in ©Vision IAS
Q 49.B
• In the beginning of the 6th century B.C., the northern India consisted of a large number of independent
kingdoms. Some of them had monarchical forms of government, while some others were republics. In
course of time, the small and weak kingdoms either submitted to the stronger rulers or gradually got
eliminated. Finally in the mid-6th century B.C., only four kingdoms – Vatsa, Avanti, Kosala and Magadha
survived.
• The Vatsa kingdom was situated on the banks of the river Yamuna. Its capital was Kausambi near
modern Allahabad. Its most popular ruler was Udayana. He strengthened his position by entering into
matrimonial alliances with Avanti, Anga and Magadha. After his death, Vatsa was annexed to the Avanti
kingdom. Hence, pair 1 is not correctly matched.
• The capital of Avanti was Ujjain. The most important ruler of this kingdom was Pradyota. He became
powerful by marrying Vasavadatta, the daughter of Udayana. He patronized Buddhism. The successors of
Pradyota were weak and later this kingdom was taken over by the rulers of Magadha. Hence, pair 2 is
correctly matched.
• Ayodhya was the capital of Kosala. King Prasenajit was its famous ruler. He was highly educated. His
position was further strengthened by the matrimonial alliance with Magadha. His sister was married to
Bimbisara and Kashi was given to her as dowry. Subsequently there was a dispute with Ajatashatru. After
the end of the conflict, Prasenjit married the daughter of Bimbisara. After the death of this powerful king,
Kosala became part of the Magadha. Hence, pair 3 is correctly matched.
• Of all the kingdoms of north India, Magadha emerged powerful and prosperous. It became the nerve
centre of political activity in north India. Magadha was endowed by nature with certain geographical and
strategic advantages. These made her to rise to imperial greatness. Her strategic position between the
upper and lower part of the Gangetic valley was a great advantage. It had a fertile soil. The iron ores in the
hills near Rajgir and copper and iron deposits near Gaya added to its natural assets. Her location at the
centre of the highways of trade of those days contributed to her wealth. Rajagriha was the capital of
Magadha. During the reign of Bimbisara and Ajatashatru, the prosperity of Magadha reached its
zenith. Hence, pair 4 is not correctly matched.
Q 50.C
• Foreign Direct Investment (FDI) is the investment through capital instruments by a person resident
outside India (other than an individual who is a citizen of Pakistan or Bangladesh or an entity that is
registered/ incorporated in Pakistan or Bangladesh).
o in an unlisted Indian company (hence, statement 1 is correct) or;
o in 10 percent or more of the post, issue paid-up equity capital on a fully diluted basis of a listed Indian
company.
• In a move that is likely to have an impact on the sale of the national carrier, the government has notified
amendments to Foreign Exchange Management Rules,2020, and allowed NRIs 100% foreign direct
investment (FDI) in Air India.
• The amendment removes the exception which permitted Overseas Citizens of India 100% FDI in air
transport, but not Air India. This category of citizens has been replaced with NRIs, now allowed to
commit 100% FDI in air transport, including Air India, through automatic route. Hence, statement 2 is
correct.
Q 51.A
• Recently around the 75th anniversary of Hiroshima being struck by the first atomic bomb to be
used in war, a district court in Japan recognised survivors of the post-explosion “black rain”, who
were outside a government-defined zone at the time of the event, as atomic bomb survivors.
• What is black rain? An estimated 69 per cent of the buildings in Hiroshima were destroyed by the
atomic bomb. The debris and soot from this, mixed with the radioactive fallout from the bomb, rose high
into the atmosphere in the form of a mushroom cloud. This material combined with the vapor in the
atmosphere and came down as dark drops of liquid that has been called black rain.
• Black rain is full of highly radioactive material, and studies have show that exposure to it can result in
serious illnesses. The rain contaminated everything it came in contact with, and dead fish were reported
floating in water bodies and severely ill cattle were seen lying in the fields. Black rain has caused acute
radiation symptoms (ARS) in many who were exposed to it, with reports of people suffering from nausea
and diarrhoea for weeks.
• The bomb dropped on Nagasaki was more powerful than the one dropped on Hiroshima, but it killed
fewer people and its effects were confined to a smaller area due to its geographical position between hills.
The blast in Nagasaki also did not produce firestorms due to the damaged area not producing enough fuel
to trigger one. This meant that the material that make up black rain was less in Nagasaki when compared
to Hiroshima, and consequently, the rain was confined to a smaller area.
17 www.visionias.in ©Vision IAS
Q 52.B
• The European Financial Stability Facility (EFSF) was created in 2010 as a temporary crisis resolution
measure in the wake of the financial and sovereign debt crisis in the euro area. The mission of the EFSF is
to safeguard financial stability in Europe by providing financial assistance to countries of the euro
area. Hence, statement 1 is not correct.
• The EFSF offered financial assistance to euro area countries in need in this context, provided they
committed to undertaking certain reforms (aimed at preventing the recurrence of similar crises). This
assistance was financed through the issuance of EFSF bonds and other capital market instruments. Hence,
statement 2 is correct.
• The EFSF does not provide any further financial assistance, as this task is now performed solely by the
ESM. Nevertheless, the EFSF continues to operate in order to:
o receive loan repayments from beneficiary countries;
o make interest and principal payments to holders of EFSF bonds;
o roll over outstanding EFSF bonds, as the maturity of loans provided to Ireland, Portugal and Greece is
longer than the maturity of bonds issued by the EFSF
• Recently, the 27-member European Union has announced EU recovery deal to counter the debilitating
effects of coronavirus on the region’s economies. Instead of individual countries raising funds, this time
around, the EU as a whole will borrow money from the markets.

Q 53.C
• Article 243M of the Indian Constitution, while exempting the Fifth Schedule areas from Part IX of the
Constitution, provides that Parliament may by law extend its provisions to the Scheduled and Tribal
Areas subject to such exceptions and modifications as may be specified in such law and no such law shall
be deemed to be an amendment to the Constitution.
• On the basis of the report of the Bhuria Committee submitted in 1995, the Parliament enacted
the Panchayats (Extension to Scheduled Areas) Act, 1996 (PESA) to extend Part IX of the Constitution
with certain modifications and exceptions to the Scheduled V areas.
• At present Scheduled V areas exist in 10 States viz. Andhra Pradesh, Chhattisgarh, Gujarat, Himachal
Pradesh, Jharkhand, Madhya Pradesh, Maharashtra, Odisha, Rajasthan and Telangana. The Ministry of
Panchayati Raj is the nodal Ministry for implementation of the provisions of PESA in the States.

Q 54.C
• The Asia-Pacific Economic Cooperation (APEC), is an economic group of 21 members, formed in
1989, with the primary goal of promoting free trade and sustainable development in the Pacific Rim
economies.
• The creation of APCE was primarily in response to the increasing interdependence of Asia-Pacific
economies. Also, the proliferation of regional economic blocs, such as the European Union (EU) and the,
now defunct, North American Free Trade Area (NAFTA), encouraged its formation.
• The founding members of APEC were Australia, Brunei, Canada, Indonesia, Japan, Korea,
Malaysia, New Zealand, the Philippines, Singapore, Thailand, and the U.S. Since its launch, China,
Hong Kong, Taiwan, Mexico, Papua New Guinea, Chile, Peru, Russia, and Vietnam have joined its
ranks. Hence option (c) is the correct answer.
• APEC refers to its members as economies rather than as states due to the disputed status of Taiwan and
Hong Kong.
Functions of APEC:
• APEC engages in multiple micro causes, like intellectual property rights and emergency preparedness,
and has many sub-groups that aim to advance policy and awareness.
• APEC has been fundamental in reducing tariffs, improving customs efficiency, and closing the gap
between developing and developed economies.

Q 55.C
• In politics, diplomacy and export control, "dual-use" refers to technology that can be used for both
peaceful and military aims. Hence option (c) is the correct answer
• The expensive technologies which would otherwise only benefit civilian commercial interests can also be
used to serve military purposes when not otherwise engaged such as the Global Positioning System.
Applications:
• Dual-use nuclear technology refers to the possibility of military use of civilian nuclear power technology.
18 www.visionias.in ©Vision IAS
• Rocket technology and intercontinental ballistic missiles
• Artificial intelligence for medical as well as military purposes examples Hololens.
• The HoloLens 2, smart glasses that allow consumers to experience augmented reality within the real
world.
• The Integrated Visual Augmented System (IVAS)
• The IVAS would be used to train soldiers, as well as field medics with battlefield experience within a
virtual environment.
• Chlorine is a chemical agent found within several household items such as Bleach and provides various
benefits with its wide array of applications. However, it can also be used as a chemical weapon.
• The image intensifiers used in night-vision goggles and the focal plane arrays found in surveillance
satellites and thermal cameras have numerous civil applications which include nature photography,
medical imaging, firefighting, and population control of predator species.

Q 56.D
• A corporate bond is a debt issued by a company in order for it to raise capital. An investor who buys a
corporate bond is effectively lending money to the company in return for a series of interest payments, but
these bonds may also actively trade on the secondary market.
• Corporate bonds are typically seen as somewhat riskier than government bonds, so they usually have
higher interest rates to compensate for this additional risk.
• A well developed corporate bond market supports economic development. It provides an alternative
source of finance and supplements the banking system to meet the requirements of the corporate
sector to raise funds for long-term investment for infrastructure.
• It is believed that this segment acts as a stable source of finance when the equity market is
volatile, and also enables firms to tailor their asset and liability profiles to reduce the risk of
maturity.
• It also helps in the diversification of risks in the system. In view of the huge investment required for the
infrastructure sector, the presence of a well developed corporate bond market assumes significance in
India. With the declining role of development finance institutions (DFIs), a developed and robust
corporate bond market becomes all the more important.
• The corporate bond market is likely to be more beneficial for business having longer-term cash flows,
where investors may be wary of risks associated with equity and long-term financing from banks may not
be easily available.
• In India, while the banks still command a sizable presence in the economy, the corporate sector is taking
recourse to the overseas markets for raising equity, debt and loans. An underdeveloped corporate bond
market can encourage this trend, thereby increasing the external sector vulnerability. Therefore a
well developed corporate bond would reduce external sector vulnerability.
• Hence, all the statements are correct.

Q 57.D
Hibernation
• It is the type of winter sleep, performed by the warm and cold-blooded animals. It is for the whole winter.
• The animals' lookout for the warmer place, their metabolic activities slow down, and it is the dormant
stage.
• Hibernation helps in maintaining the body temperature and thus avoids from any internal body damage
due to low temperatures\
• Examples - Bats, birds, mammals, insects, etc. Hence pair 1 is correctly matched.
Aestivation
• It is the type of summer sleep, performed by cold-blooded animals
• It is of shorter duration than hibernation.
• Animals search for the moist, shady and cool place to sleep.
• Aestivation helps in maintaining the body temperature by avoiding the excessive water loss and any
internal body damaged due to high temperatures.
• Examples - Bees, snails, earthworms, salamanders, frogs, earthworms, crocodiles, tortoise, etc. Hence
pair 2 is correctly matched.

19 www.visionias.in ©Vision IAS


Diapause
• It refers to the delay in development in response to regularly and recurring periods of adverse
environmental conditions.
• Diapause is most often observed in all the life stages of arthropods, especially insects. Hence pair 3 is
correctly matched.

Q 58.C
• The department of pharmaceuticals (DoP) has recently notified the production-linked incentive
(PLI) scheme to encourage domestic production of 41 active pharmaceutical ingredients (APIs), key
starting materials (KSMs) and drug intermediaries (DIs).
• Statement 1 is correct: India is a prominent and rapidly growing presence in global pharmaceuticals. It is
the largest provider of generic medicines globally, occupying a 20% share in global supply by
volume, and also supplies 62% of global demand for vaccines. India ranks 3rd worldwide for production
by volume and 10th by value.
• Statement 2 is correct: 100% Foreign Direct Investment (FDI) is allowed under the automatic route
for greenfield pharma. 100% FDI is allowed in brownfield pharma; wherein 74% is allowed under the
automatic route and thereafter through government approval route
• The Report of the Hathi Committee (1975) is an important landmark in the development of the Indian
Pharmaceutical Industry. The Hathi Committee emphasized the achievement of self-sufficiency in
medicines and of abundant availability at reasonable prices of essential medicines.
• India was self-reliant on APIs until the mid-1990s when liberalisation in import restrictions led to a
gradual influx of APIs from China. A cost-based price control system that existed until 2013 regulated the
prices of both APIs and formulations. Even though India now has a less stringent drug price control
policy, the dependence on Chinese imports has been growing. The share of China in India’s total import
of APIs has increased from 61% in 2011 to 69% in 2019.
• Drug Pricing Policy in India: The approach to price control shifted from a cost-based to a market-based
one since 2013. Now, prices of formulations are fixed based on the price of top-selling brands. The new
price control policy does not regulate the price of APIs; it regulates the prices of formulations of
those APIs, which figure in the National List of Essential Medicines (NLEM).

Q 59.B
• Recent context: A Beijing-based company which has close ties to the Chinese Communist Party has
secured exclusive development rights for the entire island of Tulagi and its surroundings.
• Tulagi Island:
o Tulagi is an island within the Solomon Islands, which are located in the South Pacific, directly
between Australia and the U.S. Hence pair 2 is correctly matched.
o It was the administrative seat (from 1893) of the British Solomon Islands Protectorate until it was
destroyed by the Japanese (1942) during World War II.
• Recent context: China has sparked a major maritime confrontation with Indonesia near the South
China Sea with dozens of Chinese fishing vessels, along with a coast guard escort, entering waters
off the Natuna Islands, which are within Indonesia’s exclusive economic zone but are also claimed
by China.
• About Natuna Island:
o The Natuna Regency is an archipelago of 272 islands located in the south part of the South China
Sea in the Natuna Sea. Hence pair 1 is correctly matched.
o It is part of the larger Tudjuh Archipelago, off the northwest coast of Borneo.
o Administratively, the islands constitute a regency within the Riau Islands Province of Indonesia.
• About Vanilla Islands:
o It is located in Southwestern Indian Ocean. Hence pair 3 is not correctly matched.
o It is an affiliation of the island nations Seychelles, Madagascar, Reunion, Mauritius, Comoros,
Mayotte in the Indian Ocean to form a new travel destination.
o Aim of the Vanilla Island that has been founded in 2010 to pool forces and jointly market the region
compared to the sole individual marketing of each island in the past.
Q 60.C
• The Consumer Protection (E-commerce) Rules, 2020 ('Rules') complement the Consumer Protection Act
2019 with regards to the protection of consumers from unfair trade practices and other practices of e-
commerce platforms.
• The new rules will be applicable to all electronic retailers (e-tailers) registered in India or abroad
but offering goods and services to Indian consumers.
20 www.visionias.in ©Vision IAS
• According to the new rules, the e-commerce players will have to display the total price of goods and
services offered for sale along with a break-up of other charges. They are also required to mention the
'expiry date' of goods offered for sale and the 'country of origin' of goods and services that are
necessary for enabling the consumer to make an informed decision at the pre-purchase stage.
• Companies are also not allowed to “manipulate the price" of goods and services offered on their platforms
to make “unreasonable profit", discriminate between consumers or make any arbitrary classification of
consumers affecting their rights under the Act.
• The Consumer Protection Act, 2019 has come into force from 20th July 2020. As provided in section
10 of the Act, the Central Consumer Protection Authority (CCPA) has been established w.e.f. 24th
July 2020. CCPA will not only protect the rights of consumers purchasing goods and services both online
and offline but also have the power to initiate class-action against companies including enforcing recalls,
refunds and returns. The violation of the rules will attract penal action under the Consumer Protection Act,
2019.
• Hence both the statements are correct.

Q 61.A
• Tolkappiyam refers to the five-fold division of lands - Kurinji (hilly tracks), Mullai (pastoral), Marudam
(agricultural), Neydal (coastal) and Palai (desert). The people living in these five divisions had their
respective chief occupations as well as gods for worship.
o Kurinji – chief deity was Murugan – chief occupation, hunting and honey collection.
o Mullai – chief deity Mayon (Vishnu) – chief occupation, cattle-rearing and dealing with dairy
products.
o Marudam – chief deity Indira – chief occupation, agriculture.
o Neydal – chief deity Varunan – chief occupation fishing and salt manufacturing.
o Palai – chief deity Korravai – chief occupation robbery.
• Hence, option (a) is the correct answer.

Q 62.A
• The ‘State of India’s Birds 2020’ report, the first comprehensive assessment of range, abundance and
conservation status of birds in India, has underlined concerns about some bird species and good news
about a few others. Released during CMS COP13, the international conference held recently in
Gandhinagar, the report was prepared as a partnership among 10 organisations including the World
Wildlife Fund, Wildlife Institute of India and Nature Biodiversity Authority-India. Much of its data
is based on citizen science — information provided by birdwatchers through various platforms.
• The number of bird species whose status was assessed. This assessment is based on three indices: long-
term trend in abundance (over 25+ years); the current annual trend in abundance (last 5 years); and
distribution range size. Hence statement 1 is correct and statement 2 is not correct.
• The national bird since 1963, the species finds itself under the highest level of legal protection, placed in
Schedule I of the Wildlife (Protection) Act, 1972 and further amendments. Peafowl is spread across plains
and hills of India, except in extremely dry or wet regions. The report finds that the abundance trend is
that of a general increase, both long term and currently.
Q 63.C
• Women poet-saints played a significant role in the bhakti movement at large. Many of these women had
to struggle for acceptance within the largely male-dominated movement. Only through demonstrations of
their utter devotion to the Divine, their outstanding poetry, and stubborn insistence of their spiritual
equality with their contemporaries were these women reluctantly acknowledged and accepted within their
ranks. Their struggle attests to the strength of patriarchal values within both societies and within religious
and social movements attempting to pave the way for more egalitarian access to the Divine.
• Women bhaktas wrote of the obstacles of home, family tensions, the absent husband, meaningless
household chores, and restrictions of married life, including their status as married women. In many cases,
they rejected traditional women’s roles and societal norms by leaving husbands and homes altogether,
choosing to become wandering bhaktas; in some instances, they formed communities with other poet-
saints. Their new focus was utter devotion and worship of their Divine Husbands.
• The Alvar women poets (Tamil poet-saints), like Andal and others, gave expression to their love for the
divine. Lal Ded (1320-1384), the Muslim poetess from Kashmir, represented the Sant tradition of
bhakti and wrote Vakhs (maxims), which are peerless gems of spiritual experience. Meera Bai, in
Gujarati, Rajasthani and Hindi (she wrote in three languages), Avvayyar, in Tamil,
and Akkamahadevi in Kannada, are well known for their sheer lyrical intensity and concentrated
emotional appeal. Their writings speak to us about the social conditions prevailing at that time, and the
21 www.visionias.in ©Vision IAS
position of woman at home and in society. They all wrote small lyrics or poems of devotional fervour,
metaphysical depth, and with a spirit of dedication and utmost sincerity. Behind their mysticism and
metaphysics is a divine madness. They turned every wound inflicted by life into a poem. Hence pairs 2
and 3 are correctly matched and 1 is not correctly matched.

Q 64.B
• Recent context: On June 14, 2019 a robot named Bumble became the first Astrobee robot to fly
under its own power in space.
• Bumble and a second Astrobee, 'Honey,' launched to the space station in April. A third robot
named 'Queen' launched in July. Hence option (b) is the correct answer.
• All about Astrobee robots:
• Astrobee is a free-flying robot system that will help researchers test new technologies in zero gravity and
perform routine work alongside astronauts aboard the International Space Station ,
• It will help astronauts reduce the time they spend on routine duties, leaving them to focus more on the
things that only humans can do.
• Working autonomously or via remote control by astronauts, flight controllers or researchers on the
ground, the robots are designed to complete tasks such as taking inventory, documenting experiments
conducted by astronauts with their built-in cameras or working together to move cargo throughout the
station.
• In addition, the system serves as a research platform that can be outfitted and programmed to carry out
experiments in microgravity - helping us to learn more about how robotics can benefit astronauts in space.
• The Astrobee system consists of three cubed-shaped robots, software and a docking station used for
recharging.
• Astrobee's navigation system uses a camera to observe the robot's surroundings and compares what the
camera sees to a map of the space station's interior.
• Astrobee robots can move in any direction and turn on any axis in space.
• The robots can fly solo, going about their business independently or they can be managed by astronauts or
flight controllers.
• Equipped with cameras, a microphone, and other sensors, they provide flying eyes and ears that will allow
flight controllers to remotely monitor conditions and remotely perform work on the station from the
ground.

Q 65.C
• Between March and May of 1947, Mountbatten decided that the Cabinet Mission Plan had become
untenable and formulated an alternative plan – referred to as the Plan Balkan.
• This plan envisaged the transfer of power to separate provinces (or to a confederation, if formed
before the transfer), with Punjab and Bengal given the option to vote for partition of their provinces.
• The various units thus formed along with the princely states (rendered independent by lapse of
paramountcy) would have the option of joining India or Pakistan or remaining separate.
• Jawaharlal Nehru strongly objected to it. He made it clear that Congress would not accept the break-up of
the Indian Union and the transfer of sovereignty to the provinces. Any plan should start with the Union
and opting out of certain areas rather than a number of sovereign units coming together if the so desired.
To jettison the basic conception of India and denude the Constituent Assembly of its significance was to
yield to the League and ignore the views of all other sections of opinion in India. The inevitable
consequences would be to invite the Balkanization of India, provoke civil conflict, and add to violence
and disorder, ensure a further breakdown of the central authority and demoralize the army, the police and
the civil service.
• Mount batten consequently abandoned the 'Plan Balkan' and started on a fresh plan by asking his advisors
to recast the whole plan to meet Nehru's main objection.
• Hence only statements 1 and 2 are correct.

Q 66.A
• The East India Company’s harsh conditions imposed on the state of Travancore, after both of them agreed
to a subsidiary alliance arrangement under Wellesley in 1805, caused deep resentment in the region. The
ruler was not able to pay the subsidy and fell in arrears. The British resident of Travancore was meddling
in the internal affairs of the state. The highhanded attitude of the Company compelled Prime Minister (or
Dalawa) Velu Thampi to rise against the Company, assisted by the Nair troops.
• Velu Thampi addressed a gathering in Kundara, openly calling for taking up arms against the British
to oust them from the native soil. This was later known as the Kundara Proclamation. There was a large
22 www.visionias.in ©Vision IAS
scale rebellion against the British as a result. A large military operation had to be undertaken to restore
peace. Hence, statement 1 is correct.
• The Maharaja of Travancore had not wholly supported the rebellion and defected to the side of the
Company. Hence, statement 2 is not correct.
• Velu Thampi killed himself to avoid capture. The rebellion petered out.

Q 67.C
• With a view to improving the liquidity position of Non-Banking Financial Companies (NBFCs) as well as
Housing Finance Companies (HFCs), the government launched a Special Liquidity Scheme of Rs. 30,000
crore. The Scheme was launched on July 1, 2020, through a Special Purpose Vehicle in the form of SLS
Trust set up by SBI Capital Markets Limited (SBICAP).
• RBI will provide funds for the Scheme by subscribing to government-guaranteed special
securities issued by the Trust. The total amount of such securities issued outstanding shall not exceed Rs.
30,000 crores at any point in time. The government of India will provide an unconditional and irrevocable
guarantee to the special securities issued by the Trust. Hence, statement 1 is correct.
• Statement 2 is not correct: Any NBFC including Microfinance Institutions registered with RBI under the
Reserve Bank of India Act, 1934 (excluding those registered as Core Investment Companies) and any
HFC registered with the National Housing Bank (NHB) under the National Housing Bank Act, 1987
which is complying with the following broad conditions will be eligible to raise funding from the said
facility:
o Compliance with RBI regulations on Capital adequacy
o Net NPA is less than 6% as on 31.03.2019
o Net profit in at least one of the two preceding financial years
o Rated as investment grade by a rating agency
o Is not reported under SMA-1 or SMA-2 category by any bank for their borrowing during the period
one year prior to 01.08.2018
• The Scheme will remain open for 3 months for making subscriptions by the Trust. The period of lending
(CPs/NCDs of NBFCs/HFCs for short duration of upto 90 days) by the Trust shall be for a period of up to
90 days. The financing would be used by the NFBCs/HFCs only to repay existing liabilities and not
to expand assets. Hence, statement 3 is correct.
Q 68.A
Non - biodegradable waste:
• Waste which cannot be degraded or broken down through microbial activities is called non-biodegradable
waste.
• Slaughterhouse residues are the result of abattoir operation in which solid and liquid wastes, as well as
wastewater, are generated in larger amounts. The principal components of the organic matter presented in
abattoir effluents are faeces, gut contents, fat, and blood. They are considered as the renewable natural
resources which can be utilized for producing biogas, fertilizer, animal feed etc.
• Such waste includes crude petroleum, plastic, styrofoam products, cans, glasses, polymers, synthetic
pesticides, radioactive fallout, some industrial effluents and metals like Lead.
• Biodegradable waste can be commonly found in municipal solid waste as green waste, food waste, paper
waste, and biodegradable plastics. Other biodegradable wastes include human waste, manure, sewage, and
slaughterhouse waste.
• Hence option (a) is correct.
Q 69.C
• Recent Context: To support the Government of India’s efforts to combat COVID-19, eVIN India is
helping the State/UT governments monitor the supply chain of COVID response material.
• Both statements 1 and 2 are correct: The Electronic Vaccine Intelligence Network (eVIN) is an
innovative technological solution aimed at strengthening immunization supply chain systems across
the country. This is being implemented under the National Health Mission (NHM) by the Ministry
of Health and Family Welfare. eVIN aims to provide real-time information on vaccine stocks and
flows, and storage temperatures across all cold chain points in the country. This robust system has
been used with the requisite customization during the COVID pandemic for ensuring the continuation of
the essential immunization services and protecting our children and pregnant mothers against vaccine-
preventable diseases.
• The Electronic Vaccine Intelligence Network has helped create a big data architecture that generates
actionable analytics encouraging data-driven decision-making and consumption-based planning that helps
in maintaining optimum stocks of vaccines leading to cost savings. Vaccine availability at all times has
increased to 99% in most health centres.
23 www.visionias.in ©Vision IAS
Q 70.A
• National Electronic Funds Transfer (NEFT) is a nation-wide centralised payment system owned and
operated by the Reserve Bank of India (RBI).
• NEFT offers the following advantages for funds transfer or receipt:
o Round the clock availability on all days of the year.
o Near-real-time funds transfer to the beneficiary account and settlement in a secure manner.
o Pan-India coverage through a large network of branches of all types of banks.
o Positive confirmation to the remitter by SMS / e-mail on credit to the beneficiary account.
o Penal interest provision for the delay in credit or return of transactions.
o No levy of charges by RBI from banks.
o No charges to savings bank account customers for online NEFT transactions.
o Besides funds transfer, NEFT system can be used for a variety of transactions including the payment
of credit card dues to the card-issuing banks, payment of loan EMI, inward foreign exchange
remittances, etc.
o Available for one-way funds transfers from India to Nepal. The outbound remittances through
NEFT system are permitted only to Nepal under Indo-Nepal Remittance. Hence, option (a) is
correct.
Q 71.C
• In 1890, Kadambini Ganguly, the first woman graduate of Calcutta University, addressed the Congress
session. This was symbolic of the fact that India's struggle for freedom would raise Indian women from
the degraded position to which they had been reduced for the past centuries. Hence option (c) is the
correct answer.
• Anandibai Joshi: was the first Indian female physician, alongside Kadambini Ganguly. She was the first
woman from the erstwhile Bombay presidency of India to study and graduate with a two-year degree
in western medicine in the United States.
• Sarojini Naidu: political activist and poet. Naidu's work as a poet earned her the sobriquet Nightingale
of India by Mahatma Gandhi. She was called Bharat Kokila by Rabindranath Tagore. She was appointed
the President of the Indian National Congress in 1925 (First Indian Woman to be appointed as President)
and later became the Governor of the United Provinces in 1947, becoming the first woman to hold the
office of Governor in the Dominion of India.
• Bina Das: was an Indian revolutionary and nationalist from West Bengal. She was a member of Chhatri
Sangha, a semi-revolutionary organisation for women in Kolkata. On 6 February 1932, she attempted to
assassinate the Bengal Governor Stanley Jackson, in the Convocation Hall of the University of
Calcutta. After her early release in 1939, Das joined the Congress party. In 1942, she participated in the
Quit India movement and was imprisoned again from 1942–45. From 1946–47, she was a member of the
Bengal Provincial Legislative Assembly and, from 1947–51, of the West Bengal Legislative Assembly.

Q 72.C
• In the First Afghan War (1839-42), the British intention was to establish a permanent barrier against
schemes of aggression from the north-west (Russia). An English army entered triumphantly into Kabul
(August 1839) after a successful attack. Most of the tribes had already been won over by bribes. Dost
Mohammed surrendered (1840) and Shah Shuja was made the Amir of Afghanistan. But Shah Shuja was
unacceptable to the Afghans. As soon as the British withdrew, the Afghans rose in rebellion, killing the
garrison commander in Kabul. The British were compelled to sign a treaty (1841) with the Afghan chiefs
by which they agreed to evacuate Afghanistan and restore Dost Mohammed.
• Under a new expedition, the British re-occupied Kabul in September 1842, but having learned their lesson
well, they arrived at a settlement with Dost Mohammed by which the British evacuated from Kabul and
recognised him as the independent ruler of Afghanistan.
• John Lawrence (1864-1869) started a policy of masterly inactivity which was a reaction to the disasters
of the First Afghan War and an outcome of practical common sense and intimate knowledge of the
frontier problem and of Afghan passion for independence.
• Even when Dost Mohammed died in 1863, there was no interference in the war of succession.
Lawrence’s policy rested on the fulfilment of two conditions:
o that the peace at the frontier was not disturbed, and
o that no candidate in civil war sought foreign help. And as Sher Ali established himself on the throne,
Lawrence tried to cultivate friendship with him.
• Hence both the statements are correct.

24 www.visionias.in ©Vision IAS


Q 73.A
• A Loya Jirga, or "grand council" in Pashto, is a mass national gathering that brings together
representatives from the various ethnic, religious, and tribal communities in Afghanistan. Hence
option (a) is the correct answer. The Loya Jirga is a centuries-old institution that has been convened at
times of national crisis or to settle national issues. Historically, it has been used to approve a new
constitution, declare war, choose a new king, or to make sweeping social or political reforms.
• According to the Afghan Constitution, a Loya Jirga is considered the "highest expression" of the Afghan
people. But it is not an official decision-making body. Its decisions are not legally binding and any verdict
it hands out must be approved by the two houses of the Afghan parliament and the president in order for it
to be made official. Unofficially, however, the Loya Jirga's decision is seen as final, with the president
and parliament expected to respect the ruling.
• The most famous gathering took place in 1747 when Pashtun tribal leaders met in the city of Kandahar to
elect a king. After nine days of debate, they chose Ahmad Shah Durrani, the man who founded
Afghanistan.
• In 2002, an Emergency Loya Jirga was held after the overthrow of the Taliban. It elected Karzai as head
of the transitional administration. It was the first time women took part in a Loya Jirga.

Q 74.C
• UN Conference on Trade and Development (UNCTAD) has released its first-ever “Digital Economy
Report 2019” that analyses the world’s digital economy. Hence option (c) is the correct answer.
• The 3 main components of the digital economy as stated in the DER 2019 include:
o Core aspects: Core aspects or foundational aspects of the digital economy comprises of fundamental
innovations (semiconductors, processors), core technologies (computers, telecommunication devices)
and enabling infrastructures (Internet and telecoms networks).
o Digital and IT sectors: It includes key products or services that rely on core digital technologies,
including digital platforms, mobile applications and payment services.
o Digitalizing sectors: It includes those products where digital products and services are being
increasingly used e.g. for e-commerce.
Key highlights of the Report:
• US and China are the top wealth creators in the digital economy, whereas Africa and Latin
American countries are lagging far behind.
• US and China account for 90% of the market capitalization value of the world’s 70 largest digital platform
companies, 75% of all blockchain technologies patents, 50% of global spending on IoT, and more than
75% of the cloud computing.
• Global Internet Protocol (IP) traffic has grown from about 100 gigabytes (GB) per day in 1992 to more
than 45,000 GB per second in 2017.
• By 2022, global IP traffic is projected to reach 150,700 GB per second.
• Estimates of the size of the digital economy range from 4.5% to 15.5% of the world’s GDP (Gross
Domestic Product).
• Regarding value-added in the information and communications technology (ICT) sector, the United States
and China together account for almost 40% of the world’s total.
• The ICT sector is the largest in Taiwan Province of China, Ireland, and Malaysia.
• The global computer services industry is dominated by the United States.
• India has the largest share among developing countries in the computer services industry.
• Also, India ranked fourth in terms of growth in the share of the ICT sector’s value-added in GDP between
2010 and 2017.
• In “Share of ICT services in total exports of services: Top 20 countries, 2017”, India ranked 2nd
which is topped by Ireland. India is the largest developing-country exporter of such services.
• Seven “super platforms” – Microsoft, Apple, Amazon, Google, Facebook, Tencent and Alibaba − account
for 2/3rd of the total market value of the top 70 platforms.

Q 75.A
• The Green Growth Equity Fund (GGEF) is a joint UK-India Fund, specifically designed to promote
sustainable energy projects. The fund, established in 2018, is focused on identifying, investing and
supporting growth in zero carbon and low carbon energy solutions in the country.
• Green Growth Equity Fund (GGEF) established with anchor investment from India’s National Investment
and Infrastructure Fund (NIIF) and the Department for International Development (DFID), Government
of UK. Hence statement 1 is correct.
25 www.visionias.in ©Vision IAS
• The Fund is based in Mumbai (India) and shall invest in scalable operating companies and platforms
across renewable energy, energy efficiency, energy storage, e-mobility, resource conservation and
associated value chains.
• GGEF aims to raise GBP 500 million from international institutional investors to invest into areas such as
renewable energy, clean transportation, water, sanitation, waste management, emerging technologies and
similar industries in India which offer significant investment opportunities and potential for attractive
returns.
• EverSource Capital, an equal joint venture between Everstone Group and Lightsource BP was selected as
the fund manager for GGEF. Hence statement 2 is not correct.

Q 76.B
• Pattachitra style of painting is one of the oldest and most popular art forms of Odisha. The name
Pattachitra has evolved from the Sanskrit words patta, meaning canvas, and Chitra, meaning picture.
Pattachitra is thus a painting done on canvas and is manifested by rich colourful application, creative
motifs and designs, and portrayal of simple themes, mostly mythological in depiction. Hence statement 1
is correct.
• Making the patta is the first thing that comes in the agenda, and the painters, also called chitrakars, go
about their work in preparing a tamarind paste, which is made by soaking tamarind seeds in water for
three days. The seeds are later pounded with a crusher, mixed with water, and heated in an earthen pot to
turn it to a paste, which is called niryas kalpa. The paste is then used to hold two pieces of cloth
together with it, and coated with a powder of soft clay stone a couple of times till it becomes
firm. Soon as the cloth becomes dry, the final touch of polishing it with a rough stone and then a smooth
stone or wood is given, until the surface becomes smooth and leathery, and is all ready as a canvas to be
painted on. Hence statement 2 is correct.
• Preparing the paints is perhaps the most important part of the creation of Pattachitra, engaging the
craftsmanship of the chitrakars in using naturally available raw materials to bring about indigenous
paints. The gum of the kaitha tree is the chief ingredient, and is used as a base for making different
pigments, on which diverse raw materials are mixed for diverse colours. Powdered conch shells, for
instance, are used for making a white pigment, while lamp soot is used for a black pigment. The root of
the keya plant is usually used for making the common brush, while mouse hair is used on the requirement
of finer brushes, to be attached to wooden handles.
• The creation of the Pattachitra paintings is a disciplined art form, and the chitrakars maintain rigidity in
their use of colours and patterns, restricting the colours to a single tone. Hence statement 3 is not
correct.

Q 77.B
• Recent context: Researchers have developed an Artificial Intelligence-powered tool that has been
trained to “look” at color images and identify galaxy clusters quickly. Hence option (b) is the
correct answer.
About DEEP-CEE (Deep Learning for Galaxy Cluster Extraction and Evaluation.)
• Deep-CEE builds on Abell’s approach for identifying galaxy clusters.
• It replaces the astronomer with an Artificial Intelligence model.
• Deep-CEE model is based on neural networks.
• These neural networks are designed to mimic the way a human brain learns to recognize objects by
activating specific neurons when visualizing distinctive patterns and colors.
• Artificial Intelligence models are repeatedly trained until the algorithm is able to learn to associate objects
on its own.
Galaxy clusters:
• They are some of the most massive structures in the cosmos, consisting of hundreds of galaxies
spread in not so vast area, usually it is limited to a few million light-years across.
• At their core, these clusters have big population of elliptical yet massive galaxies.
• These clusters are believed to be formed stars a long time ago but not making stars anymore.

Q 78.A
• Temperatures at the earth's surface have increased by between 0.2 and 0.4 degrees C in the past 30 years.
• This warming trend is attributed to higher concentrations of greenhouse gases – CO2, methane, CFCs, and
others – which warm both the earth's surface and lower atmosphere by holding heat in. Most of the
atmospheric greenhouse gases are concentrated in the upper troposphere. The lower stratosphere,
26 www.visionias.in ©Vision IAS
approximately between six and 30 miles above the Earth’s surface, is mostly above the layer of
greenhouse gases that trap heat. Hence, statement 1 is not correct.
• Temperatures decrease with altitude in the troposphere at a rate of about 6.5 degrees Celsius per
kilometer, depending on the weather. While in stratosphere, temperature increase with altitude. It is
because the ozone layer absorbs the greater part of the solar ultraviolet radiation. The ozone layer is an
absorbing agent that protects life on Earth. Hence, statement 2 is correct.

Q 79.D
• Getting back the required level of groundwater is called recharging of groundwater. For this, the
water accumulated on the earth-surface is facilitated to enter into the earth and to raise the level of
groundwater.
• Some technologies applied for the recharging of groundwater are:
Construction of check dams:
• A permanent or temporary wall constructed against the flow of water current in order to check it
against going away and also in order to raise the existing water level is called as a check
dam. Temporary check dams are constructed by using different types of locally available materials like
wood, pebbles, rock pieces, or wire nets.
Construction of Irrigation Ponds:
• These are artificially constructed ponds in the area of watersheds for the collection of water for irrigation.
This practice is very much popular in South India.
Construction of Percolation Ponds or Percolation Pits:
• These are artificially constructed ponds in low lying areas to collect the rainwater. Invertedtube wells and
dug wells are constructed in the bottoms of ponds so as to allow easy percolation of water inside the
ground and to recharge the groundwater level.
Construction of Tunnels:
• These are deep structures in the ground, dug in order to collect the rainwater in them. The rainwater so
collected, in tunnels, recharges the groundwater table.
Sub strata-Dykes or Subsurface dams:
• These are long thick walls built undergrounds in the bed of some streams to stop flooding on to a low area
of land or checking the sub-surface flow of water.
Inverted Tube wells:
• These are deep bore wells through which the rainwater is allowed to enter into the earth to recharge the
groundwater. Tube wells are used to extract the groundwater.
Gully Plugs:
• The deep incision that cuts into the bedrock due to surface runoff of water is called a gully. Gullies
are formed in steep areas due to the erosion of soil by rainwater. Gully plugs are stones, sandbags or wood
logs that are used as obstructions against the flow of water through gullies.
• Hence option (d) is the correct answer.

Q 80.C
• Recently the Indian Railways has initiated the process to allow private firms to operate passenger trains on
its network by inviting entities to participate on 109 origin-destination routes through 151 new trains. In
return, private operators will have to pay fixed haulage charges, energy charges on the basis of actual
consumption, and a share of their gross revenue to the Railways.
• The FDI Policy permits 100% FDI in railways infrastructure sector. FDI is permitted in the
construction, operation and maintenance of the railway transport sector: Suburban corridor projects
through PPP model, High-speed train projects, Dedicated freight lines, Rolling stock including train sets,
and locomotives/coaches manufacturing and maintenance facilities, Railway electrification, Signalling
systems, Freight terminals and Passenger terminals. Hence statement 1 is correct.
• Indian Railway Catering and Tourism Corp. Ltd (IRCTC), a subsidiary of Indian Railways has launched
brand new Tejas Express (passenger) trains, an initiative that will help the state-run company generate
additional revenue and plan for more such offerings. This is the first time that IRCTC, which manages the

27 www.visionias.in ©Vision IAS


bookings and catering services on Indian Railways’ 120,000 km network, will be managing train services
on its own. IRCTC will have the rights to advertise on these trains to earn non-fare revenues. The new
Tejas trains will be the first to be run by an entity other than the national transporter. Hence
statement 2 is correct.
• The Railway Board had constituted a Committee for mobilization of resources for major railway projects
and restructuring of Railway Ministry and Railway Board under the Chair of Mr. Bibek Debroy. The
committee submitted its report in June 2015. The Committee recommended large scale private
participation.

Q 81.B
• Specific Constitutional provisions related to water:
o Union List: Regulation and development of inter-state rivers and river valleys. Hence statement
1 is not correct.
o State List: Water that is to say, water supplies, irrigation and canals, drainage and embankments,
water storage and water power subject to the provisions of Union List.
• Article 262 of the Constitution deals with adjudication of water disputes. The provisions in this regard
are:
o Article 262 (1): Parliament may, by law, provide for the adjudication of any dispute or complaint with
respect to the use, distribution or control of the waters of, or in, any inter-State river or river valley.
o Article 262 (2): Parliament may, by law, provide that neither the Supreme Court nor any other
court shall exercise jurisdiction in respect of any such dispute. Hence statement 2 is correct.
• Under Article 262, the “Inter-State River Water Disputes Act, 1956” was enacted by the Parliament.
Till date, Nine Inter-State River Water Disputes Tribunals were constituted.
• Recently The Inter-State River Water Disputes (Amendment) Bill, 2019 was passed in Lok Sabha.
Some of the important provisions:
o Disputes Resolution Committee: Under the Bill, when a state puts in a request regarding any water
dispute, the central government will set up a Disputes Resolution Committee (DRC), to resolve the
dispute amicably.
o Tribunal: The central government will set up an Inter-State River Water Disputes Tribunal, for the
adjudication of water disputes.
o Time frames: Under the Act, the Tribunal must give its decision within three years, which may be
extended by two years.
o The Decision of the Tribunal has the same force as that of an order of the Supreme Court.
o Data bank: Under the Act, the central government maintains a data bank and information system at
the national level for each river basin.

Q 82.A
• In order to address the need for building professionally managed and shared science and technology
infrastructure in the country, the Department of Science & Technology has launched a unique scheme
called ‘Sophisticated Analytical & Technical Help Institutes’ (SATHI). It aims to build infrastructure
which is readily accessible to academia, start-ups, manufacturing, industry and R&D labs. Hence
option (a) is the correct answer.
• These Centres are expected to house major analytical instruments to provide common services of high-end
analytical testing, thus avoiding duplication and reduced dependency on foreign sources. These would be
operated with a transparent open-access policy. DST has already set up three such centres in the country,
one each at IIT Kharagpur, IIT Delhi and Banaras Hindu University at the cost of Rs 125 crore each.

Q 83.A
• In a resumption of bilateral and multilateral military exercises which were deferred due to
coronavirus (COVID-19), India will take part in the Russian Kavkaz 2020 strategic command-post
exercise. Hence option (a) is the correct answer. The invitees for the exercise also include China and
Pakistan apart from other member-states of the Shanghai Cooperation Organisation (SCO).
• The invitation for participation to Kavkaz 2020, also referred to as Caucasus-2020, has been extended to
at least 18 countries including China, Iran, Pakistan and Turkey apart from other Central Asian Republics
part of the SCO. In June 2020, India and Chinese military contingents marched at the Victory Day Parade
at Red Square in Moscow to mark the 75th anniversary of World War II.

28 www.visionias.in ©Vision IAS


Q 84.B
• The National Centre for Polar and Ocean Research, (NCPOR) formerly known as the National Centre
for Antarctic and Ocean Research (NCAOR) is an Indian research and development institution, situated in
Vasco da Gama, Goa.
• NCPOR was established as an autonomous Research and Development Institution of the Ministry
of Earth Sciences (formerly Department of Ocean Development), Government of India on the 25th May
1998. Hence statement 1 is not correct.
• It is responsible for administering the Indian Antarctic Programme and maintains the Indian
government's Antarctic research stations, Bharati and Maitri and also operating the Himadri and
IndARC Arctic research stations in Svalbard, Norway. Hence statement 2 is correct.
• The NCPOR operates in different fields or tasks:
o storing ice core samples, from Antarctica and the Himalayas.
o managing the oceanic research vessel ORV Sagar Kanya, the flagship of India's fleet of
oceanographic study vessels. This ship has contributed significantly to India's study of the Arabian
Sea, the Bay of Bengal, and the Indian Ocean.
Q 85.A
• The shore temple at Mahabalipuram was built in the reign of Narasimhavarman II, also known as
Rajasimha who reigned from 700 to 728 CE. It houses three shrines, two to Shiva, one facing east
and the other west, and a middle one to Vishnu who is shown as Anantashayana. This is unusual,
because temples generally have a single main shrine and not three areas of worship. This shows that it was
probably not originally conceived like this and different shrines may have been added at different times,
modified perhaps with the change of patrons. Hence option (a) is the correct answer.
• In the compound there is evidence of a water tank, an early example of a gopuram, and several other
images. Sculptures of the bull, Nandi, Shiva’s mount, line the temple walls, and these, along with the
carvings on the temple’s lower walls have suffered severe disfiguration due to erosion by salt-water laden
air over the centuries.
• Brihadishvara Temple, also called Rajarajesvaram is a temple dedicated to Shiva located in South bank
of Kaveri river in Thanjavur, Tamil Nadu, Built by Tamil king Raja Raja Chola I between 1003 and 1010
AD, the temple is a part of the UNESCO World Heritage Site known as the "Great Living Chola
Temples", along with the Chola dynasty era Gangaikonda Cholapuram temple and Airavatesvara temple
• The Kailashanatha temple is the largest of the rock-cut Hindu temples at the Ellora Caves, Maharashtra,
Most of the excavation of the temple is generally attributed to the eighth century Rashtrakuta king Krishna
I with some elements completed later.
• Virupaksha Temple is located in Hampi in the Ballari district of Karnataka, India. It is part of the Group
of Monuments at Hampi, designated as a UNESCO World Heritage Site. The temple is dedicated to Lord
Virupaksha, a form of Shiva. The temple was built by Lakkan Dandesha, a nayaka (chieftain) under the
ruler Deva Raya II of Viajaynagar Empire.

Q 86.C
• External Commercial Borrowings are commercial loans raised by eligible resident entities from
recognised non-resident entities and should conform to parameters such as minimum maturity, permitted
and non-permitted end-uses, maximum all-in-cost ceiling, etc.
• All entities eligible to receive FDI are also eligible to raise ECBs. Further, the following entities are also
eligible to raise ECB:
o Port Trusts;
o Units in SEZ;
o SIDBI;
o EXIM Bank; and
o Registered entities engaged in micro-finance activities, viz., registered Not for Profit companies,
registered societies/trusts/cooperatives.
• The negative list, for which the ECB proceeds cannot be utilised, would include the following:
o Real estate activities (However, this would not include construction/development of industrial
parks/integrated township/SEZ, purchase/long term leasing of industrial land as part of new
project/modernisation of expansion of existing units or any activity under ‘infrastructure
sector’ definition).
o Investment in the capital market.
o Equity investment.
o Working capital purposes except from foreign equity holder.
o General corporate purposes except from foreign equity holder.
29 www.visionias.in ©Vision IAS
o Repayment of Rupee loans except from foreign equity holder.
o On-lending to entities for the above activities.
• Hence, option (c) is correct answer.

Q 87.D
• Like any other opaque objects illuminated by a light source, the Moon and the Earth cast shadows into
space as they block the sunlight that hits them. Each shadow has 3 different areas: the umbra, the
penumbra, and the antumbra.
o The penumbra is the lighter outer part of a shadow. The Moon's penumbra causes partial solar
eclipses, and the Earth's penumbra is involved in penumbral lunar eclipses.
o Umbra is the shadow's dark center portion.
o Antumbra is the lighter part of the shadow that begins where the umbra ends.
• When the Earth enters the Moon's shadow, we see a solar eclipse; when the Moon travels through the
Earth's shadow, a lunar eclipse occurs. The type of eclipse depends on the type of shadow that is involved
• A penumbral lunar eclipse occurs when Earth's outer shadow falls on the Moon's face. It is easy to miss
because most of the Moon is illuminated by sunlight.
• During a penumbral lunar eclipse, the Moon does not disappear partially or fully, it just becomes a bit
faint, which means that humans might not be able to differentiate between the full Moon and the eclipsed
Moon.
• It recently occurred on July 5. The eclipse was only visible from South/West Europe, much of Africa,
much of North America, South America, Pacific, Atlantic, Indian Ocean and Antarctica. It was not visible
from India this time as it will be day time during the celestial event in the country. Hence, statement 1 is
not correct.
• It can occur more than once in a year. The next penumbral lunar eclipse will take place between
November 29 to November 30. Hence, statement 2 is not correct.

Q 88.B
• A super-Earth is an extrasolar planet with a mass higher than Earth's, but substantially below
those of the Solar System's ice giants, Uranus and Neptune, which are 14.5 and 17 times Earth's,
respectively. These planets can be up to 10 times more massive than Earth. If the super-Earth is in the
habitable zone of its star and has a life-friendly atmosphere, it could host life, but that is not a guarantee.
• Also called the habitable zone or life zone, the Goldilocks region is an area of space in which a planet is
just the right distance from its home star so that its surface is neither too hot nor too cold. The term
"Super-Earth" refers only to the mass of the planet, and so does not imply anything about the surface
conditions or habitability. Hence statement 1 is not correct.
• There are no known super-Earths in our solar system; they are all in extrasolar planetary systems.
With telescope technology of today, most super-Earths have been found near M-class red dwarf stars
(smaller than the sun) or G-class stars (similar to our sun). Stars that are bigger and shine brighter than the
sun would simply wash super-Earths out, and make their tiny gravitational tugs hard to detect and the
transit signals too small. Hence statement 2 is correct.
Q 89.A
• Plea bargaining refers to a person charged with a criminal offence negotiating with the prosecution
for a lesser punishment than what is provided in law by pleading guilty to a less serious offence. It is
common in the United States, and has been a successful method of avoiding protracted and complicated
trials. As a result, conviction rates are significantly high there. It primarily involves pre-trial negotiations
between the accused and the prosecutor. It may involve bargaining on the charge or in the quantum of
sentence. Hence statement 1 is correct.
• Plea bargaining was introduced in 2006 as part of a set of amendments to the CrPC as Chapter XXI-A,
containing Sections 265A to 265L.
• The Indian code makes plea bargaining a process that can be initiated only by the accused; further,
the accused will have to apply to the court for invoking the benefit of bargaining. Hence statement 2 is
not correct.
• Cases for which the practice is allowed are limited. Only someone who has been charge sheeted for an
offence that does not attract the death sentence, life sentence or a prison term above seven years can
make use of the scheme under Chapter XXI-A. Hence statement 3 is not correct.
• It is also applicable to private complaints of which a criminal court has taken cognisance. Other categories
of cases that cannot be disposed of through plea bargaining are those that involve offences affecting the
“socio-economic conditions” of the country, or committed against a woman or a child below the age of
14.
30 www.visionias.in ©Vision IAS
Q 90.D
• Once an industry is accorded infrastructure status, the sector is entitled to a number of benefits and
concessions.
• This grant of status enables the industry to raise money from insurance companies, pension funds, and
international lenders with longer tenure and on easier terms.
• Infrastructure status gives industries access to cheaper foreign currency funding through the external
commercial borrowing route.
• Falling under the infrastructure category helps the sector get credit at competitive rates and on a long-
term basis with enhanced limits. Furthermore, the logistics sector will also be eligible to borrow
from the India Infrastructure Financing Company (IIFCL).
• According to infrastructure status will further simplify the approval process for affordable projects,
create clear guidelines, and increase transparency in the segment.
• For example, Logistics has been accorded “infrastructure” status in 2017, facilitating the availability of
debt on easier terms and access to external commercial borrowings, longer tenor funds from insurance
companies, and pension funds, as well as investment support. Hence, all the options are correct.

Q 91.B
• Recent context: India has been ranked 112th among 153 countries in the annual Global Gender
Gap Index for 2020, published by the World Economic Forum (WEF).
About Gender Gap Report:
• The WEF published its first gender gap index in 2006.
• The Global Gender Gap Index benchmarks 153 countries on their progress towards gender parity in four
dimensions:
o Economic Participation and
o Opportunity, Educational Attainment,
o Health and Survival and
o Political Empowerment. Hence option (b) is the correct answer.
• The Report aims to serve as a compass to track progress on relative gaps between women and men on
health, education, economy and politics.
• It measures women’s disadvantage compared to men, and is not a measure of equality of the gender gap.
• Iceland, Norway, and Finland occupy the top three spots in the Report.
• India’s position in the Index:
o India has slipped four places in the report to 112, behind neighbours China, Sri Lanka, Nepal
and Bangladesh.
o It is due to due to rising disparity in terms of women’s health and participation in the economy.
o The country ranked 98th in WEF’s first report in 2006. Since then, it has fallen due to poor
performance in three out of four indicators.
o India is also ranked in the bottom-five in terms of women’s health and survival and economic
participation.
o The report showed that economic opportunities for women are extremely limited in India (35.4
per cent).
o India also ranked among countries with very low women representation on company boards.
The report highlighted abnormally low sex ratios at birth in India (91 girls for every 100 boys).
o On health and survival, four large countries like Pakistan, India, Vietnam and China fare badly
with millions of women not getting the same access to health as men.
o India is the only country among the 153 countries studied where the economic gender gap is larger
than the political one.
o On a positive note, India has closed two-thirds of its overall gender gap.

Q 92.B
• Recent context: The United Nations Children's Fund (UNICEF) launched the ‘YuWaah’ also
known as Generation Unlimited Programme in India on November 1, 2019.
• According to UNICEF, Generation Unlimited Programmer, called YuWaah in India, is a multi-
stakeholder alliance which aims to facilitate youth to gain relevant skills for productive lives and the
future of work. It aims to ensure that every young person age 10-24 is in some form of learning,
school, self-employment, training, or age-appropriate employment by 2030. Hence option (b) is the
correct answer.
• The target age group of YuWaah includes adolescent girls and boys.
31 www.visionias.in ©Vision IAS
• Its key mission is to promote access to foundational, transferable and 21st century skills for youth inside
and outside formal education systems, which includes defining foundational skills, life skills and flexible
learning and identifying and scaling impactful delivery models.
• YuWaah intends to create platforms to guide youth to market opportunities (career guidance, mentorship,
internships, apprenticeships) and facilitate integration of career guidance in school education.

Q 93.C
• India is the largest emitter of sulphur dioxide (SO2) in the world, contributing more than 15 per cent of
global anthropogenic emissions, according to a new report by Greenpeace.
• The primary reason for India’s high emission output is the expansion of coal-based electricity generation
over the past decade, the report added. Five of the top 10 SO2 emission hotspots from coal/power
generation industry across the world are in India.
• SO2 emissions are a significant contributor to air pollution. Its direct exposure and exposure to particulate
matter PM2.5 (fine particulate matter) produced when SO2 reacts with other air pollutants to form
sulphate particles both affect human health.
• The greatest source of SO2 in the atmosphere is the burning of fossil fuels in power plants and other
industrial facilities. Other sources include industrial processes such as extracting metal from ore, natural
sources such as volcanoes, and locomotives, ships and other vehicles and heavy equipment that burn fuel
with high sulphur content. Hence statement 1 is correct.
• Flue gas desulfurization (FGD) is a set of technologies used to remove sulfur dioxide (SO2) from
exhaust flue gases of fossil-fuel power plants, as well as from the emissions of other sulfur oxide
emitting processes. Hence statement 2 is correct.
• The Union Ministry of Environment, Forest and Climate Change had, for the first time, introduced SO2
emission limits for coal-fired power plants in December 2015. But a Supreme Court order changed the
deadline for installation of FGD technology in power plants from 2017 to December 2019 in Delhi-NCR
and till 2022 for other parts of the country.
• Recently, the Supreme Court rejected the request of the Association of Power Producers (APP) to extend
the 2022 deadline for meeting pollution norms by two years.

Q 94.C
• The Government of India has instituted a voluntary scheme for labelling of environment-friendly products
to be known as ECO Mark in 1991. Hence, statement 1 is correct.
• The scheme is being administered by the Bureau of Indian Standards. Hence, statement 2 is correct.
• So far the following product categories have been identified for coverage under this scheme: a) Soaps and
Detergents; b) Paints c) Paper d) Plastics e) Cosmetics f) Textiles g) Batteries h) Wood Substitutes j)
Propellants and Aerosols k) Food Items (edible oils – including Vanaspati, Tea and Coffee) m) Electrical
and Electronics Goods n) Packing/Packaging Materials p) Lubricating/Speciality Oils q) Drugs r) Foods
Preservatives and Additives s) Pesticides t) Leather
• The Scheme is being operated on a national basis and provides certification and labeling for household
and other consumer products which meet certain environmental criteria along with quality requirements
prescribed in relevant Indian Standards for the product.

Q 95.D
• Multilateral environmental agreements guide global, regional and national action on environmental issues
and are a result of multilateral processes, which make them key elements of environmental, legal and
governance regimes.
• Scholars and practitioners also refer to them as “soft laws” to indicate the nature of the instruments and
compliance issues related to them. Some of the important Multilateral Environmental Agreements ratified
by India are:
o Convention on Biological Diversity (CBD): It is the international legal instrument for “the
conservation of biological diversity, the sustainable use of its components and the fair and equitable
sharing of the benefits arising out of the utilization of genetic resources”. CBD has been ratified by
196 nations. India ratified the Convention on Biological Diversity in February 1994.
o United Nations Convention to Combat Desertification (UNCCD): United Nations Convention to
Combat Desertification (UNCCD) is the sole legally binding international agreement linking
environment and development to sustainable land management. The Convention addresses
specifically the arid, semi-arid and dry sub-humid areas, known as the drylands, where some of the
most vulnerable ecosystems and peoples can be found. India ratified the UNCCD Convention on 17th
December 1996.
32 www.visionias.in ©Vision IAS
o The International Treaty on Plant Genetic Resources for Food and Agriculture: Also known as
Seed Treaty as it is a comprehensive international agreement for ensuring food security through the
conservation, exchange and sustainable use of the world's Plant Genetic Resources for Food and
Agriculture. India is a signatory to the treaty.
• Other important Environmental Agreements and global environmental conventions ratified by India
include:
o Convention on International Trade in Endangered Species of Wild Fauna and Flora (CITES)
o The Wildlife Trade Monitoring Network (TRAFFIC)
o Convention on the Conservation of Migratory Species of Wild Animals
o Convention on Wetlands (popularly known as the Ramsar Convention)
o Minamata Convention on Mercury
o United Nations Framework Convention on Climate Change (UNFCCC)
o Montreal Protocol (on Ozone Depleting Substances)
o Stockholm Convention on Persistent Organic Pollutants (POPs)
o Rotterdam Convention on Prior Informed Consent (PIC) for certain Hazardous Chemical and
Pesticides in International Trade.
o Basel Convention on the Control of Trans-boundary Movement of Hazardous Waste and Their
Disposal.

Q 96.A
• Article 351 of the Constitution speaks of the development of Hindi as the Official Language of the
Union.
• In 1963, the Official Languages Act was enacted. It provided for the setting up of a Committee on
Official Language to review the progress made in the use of Hindi for the official purpose of the
Union. Hence statement 1 is correct.
• The Committee consists of thirty members, of whom twenty shall be members of the House of the
people and ten shall be members of the Council of States to be elected respectively by the members of
the House of the People and the members of the Council of States in accordance with the system of
proportional representation by means of the single transferable vote.
• The Chairman of the Committee is elected by the members of the Committee. As a convention, the
Union Home Minister has been elected as Chairman of the Committee from time to time. Hence
statements 2 is not correct.
• It is the duty of the Committee to review the progress made in the use of Hindi for the Official purposes of
the Union and submit a report to the President making recommendations thereon and the President
shall cause the report to be laid before each House of Parliament and sent it to all the State
Governments. Hence statement 3 is correct.

Q 97.C
The World Health Organisation (WHO) has undertaken various initiatives to address Anti-Microbial
Resistance:
• Global Antimicrobial Resistance Surveillance System (GLASS): supports a standardized approach to
the collection, analysis and sharing of data related to antimicrobial resistance at a global level.
• AWaRE tool: aimed at guiding policy-makers and health workers to use antibiotics safely and more
effectively. It classifies antibiotics into three groups:
o Access — antibiotics used to treat the most common and serious infections
o Watch — antibiotics available at all times in the healthcare system
o Reserve — antibiotics to be used sparingly or preserved and used only as a last resort. Hence
the correct option is (c).
• Global Antibiotic Research and Development Partnership (GARDP): encourages research and
development through public-private partnerships.
• Interagency Coordination Group on Antimicrobial Resistance (IACG): established by United Nations
Secretary-General to improve coordination between international organizations and to ensure effective
global action.
• Global Action Plan aims to ensure prevention and treatment of infectious diseases with safe and effective
medicines.

33 www.visionias.in ©Vision IAS


Q 98.C
• The World Climate Research Programme (WCRP) mission is to facilitate the analysis and prediction of
Earth system variability and change for use in an increasing range of practical applications of direct
relevance, benefit and value to society. The two overarching objectives of the WCRP are:
o to determine the predictability of climate; Hence statement 1 is correct.
o to determine the effect of human activities on climate
• WCRP has three co-sponsors:
o World Meteorological Organization: WMO is a specialized agency of the United Nations, which is
dedicated to international cooperation and coordination on the state and behaviour of the Earth’s
atmosphere, its interaction with the land and oceans, the weather and climate it produces, and the
resulting distribution of water resources. WMO also hosts the WCRP Secretariat. Hence statement 2
is correct.
o Intergovernmental Oceanic Commission of UNESCO: IOC-UNESCO is the United Nations
marine science organization. The purpose of the Commission is to promote international cooperation
and to coordinate programmes in research, services and capacity-building, in order to learn more
about the nature and resources of the ocean and coastal areas and to apply that knowledge for the
improvement of management, sustainable development, the protection of the marine environment, and
the decision-making processes of its Member States.
o International Science Council: ISC was created in 2018 as the result of a merger between the
International Council for Science (ICSU) (previously a sponsor of WCRP) and the International
Social Science Council (ISSC). It is the only international non-governmental organization bringing
together the natural and social sciences and the largest global science organization of its type with a
vision to advance science as a global public good.

Q 99.D
• The FATF is the global money laundering and terrorist financing watchdog. The inter-governmental
body sets international standards that aim to prevent these illegal activities and the harm they cause to
society. Hence statement 1 is correct.
• It was established in July 1989 by a Group of Seven (G-7) Summit in Paris, initially to examine and
develop measures to combat money laundering. Hence statement 2 is correct.
• It currently comprises 37 member countries (including India) and 2 regional organizations-
European Commission and Gulf Co-operation Council. Hence statement 3 is correct.
• Role of FATF in combating terrorist financing:
• Setting global standards to combat terrorist financing.
• Evaluating countries’ ability to prevent, detect, investigate and prosecute the financing of
terrorism: FATF issues two lists namely-
• Black list (officially known as High-Risk Jurisdictions subject to a Call for Action)
o It sets out the countries that are considered deficient in their anti-money laundering and counter-
financing of terrorism (AML/CFT) regulatory regimes.
o These blacklisted countries are subjected to economic sanctions and other prohibitive measures by
FATF member states and other international organizations.
o The current FATF blacklist includes two countries: North Korea and Iran.
• Grey list (officially referred to as Jurisdictions Under Increased Monitoring)
o Countries on the FATF grey list represent a much higher risk of money laundering and terrorism
financing but have formally committed to working with the FATF to develop action plans that will
address their AML/CFT deficiencies.
o These countries are subjected to increased monitoring by the FATF.
o While grey-list classification is not as negative as the blacklist, countries on the list may still face
economic sanctions from institutions like the IMF and the World Bank and experience adverse effects
on trade.
• Assisting jurisdictions in implementing financial provisions of the United Nations Security Council
resolutions on terrorism

Q 100.B
• With an aim to make India a commercial arbitration hub and reduce the intervention of courts, the
Government introduced amendments to the Arbitration and Conciliation Act, 1996. The Arbitration and
Conciliation (Amendment) Act, 2019 stems from the recommendations made by a high-level committee
under the chairmanship of Justice BN Srikrishna.

34 www.visionias.in ©Vision IAS


• Important Changes brought about by the Amendment:
o Establishment of ACI: It will establish an independent body, namely the Arbitration Council of
India (‘ACI’) that will frame policies, provide grading of arbitral institutions, and oversee the timely
and cost-effective disposal of arbitration cases. The statutory body is to be chaired by a judge of the
Supreme Court, or the Chief Justice of a High Court or an eminent person, having special knowledge
and experience in the conduct of the arbitration.
o The Council will make grading of arbitral institutions on the basis of criteria relating to infrastructure,
quality and caliber of arbitrators, performance and compliance of time limits for disposal arbitrations.
Hence statement 3 is correct.
o Time frame: Section 29 A of the Act which requires completion of all the arbitral proceedings
within one year of the arbitral tribunal being constituted. However, it excludes International
Commercial Arbitration from the ambit of the section. Hence statement 2 is not correct.
o The Amendment Act empowers the Supreme Court (in the case of international commercial
arbitration) and the High Court (in cases other than international commercial arbitration) to
designate arbitral institutions for the purpose of appointment of arbitrators. It also provided for
the qualifications and experience of Arbitrators. Hence statement 1 is not correct.
o The arbitrator, the arbitral institution and the parties to the arbitration agreement must maintain
the confidentiality of all arbitral proceedings.

Copyright © by Vision IAS


All rights are reserved. No part of this document may be reproduced, stored in a retrieval system or transmitted
in any form or by any means, electronic, mechanical, photocopying, recording or otherwise, without prior
permission of Vision IAS.

35 www.visionias.in ©Vision IAS

You might also like